• Skip to content

GMAT Prep Courses & Tutoring

GMAT AWA Topics – Sample Prompts

Before you start.

Remember that the AWA is testing your ability to effectively critique an argument being presented. There will be some common fallacies used in the prompts that you will need to address. Familiarize yourself with these flawed arguments so that you can quickly identify them as you write your essay:

Inappropriate Comparisons – comparing two things which are not necessarily similar, thus leading to a flawed conclusion. You will notice that in an inappropriate comparison, the argument will only note the similarities between two things with no explanation of why the differences do not change the outcome.

Example: "Dogs are highly motivated by food, which is a useful tool for training. Using this same method, you can help to encourage your children to repeat positive behaviors." This argument does not address early childhood development, nor does it address dog psychology – the person assumes that if a solution worked in one situation, it will work in the other.

Vague Language – using words such as "many", "few", "some", "more", or "less" without further qualification is often too vague to make a cogent point.

Example: "In Chicago, more people voted to add bike lanes than people did in Aurora, so it is clear that the citizens in Chicago are more pro-bike than the citizens in Aurora." This argument does not explain what 'more' means. Is it by percentage? Does it refer to raw number of votes, and if so, does it take into account the number of voters in Chicago vs. the number of voters in Aurora?

Correlation vs Causation – this common fallacy asserts that because something occurred, it must be a consequence of some specific variable observed before the result.

Example: "Students in Neighborhood A have higher test scores on average that students in Neighborhood B. Therefore, it is clear that the schools in Neighborhood A are better than the schools in Neighborhood B." This argument is flawed because it assumes the only meaningful variable in test scores was the school the children attend. This argument does not consider other various factors, such as poverty, school resources, home situations, and the parents' abilility to supplement educational opportunities, etc.

Sampling Issues – a case in which statistics are used inappropriately to compare unlike populations or to draw broad conclusions using a small sample.

Example: "56% of the reviews of Company A are negative, which shows that most customers are very unhappy with the service provided." The problem with this argument is that it does not address the percentage of overall customers who have written reviews. How do we know that this is representative of all customers vs. those who are upset about an issue and take the time to write a review?

Now that you have reviewed some common flawed arguments you may encounter, it is time to get started practicing!

Sample Prompt 1

The following appeared in the health section of a magazine on trends and lifestyles:

"People who use the artificial sweetener aspartame are better off consuming sugar, since aspartame can actually contribute to weight gain rather than weight loss. For example, high levels of aspartame have been shown to trigger a craving for food by depleting the brain of a chemical that registers satiety, or the sense of being full. Furthermore, studies suggest that sugars, if consumed after at least 45 minutes of continuous exercise, actually enhance the body’s ability to burn fat. Consequently, those who drink aspartame-sweetened juices after exercise will also lose this calorie-burning benefit. Thus, it appears that people consuming aspartame rather than sugar are unlikely to achieve their dietary goals."

Discuss how well reasoned you find this argument. In your discussion, be sure to analyze the line of reasoning and the use of evidence in the argument. For example, you may need to consider what questionable assumptions underlie the thinking and what alternative explanations or counterexamples might weaken the conclusion. You can also discuss what sort of evidence would strengthen or refute the argument, what changes in the argument would make it more logically sound, and what, if anything, would help you better evaluate its conclusion.

Sample Prompt 2

The following appeared in the opinion column of a financial magazine:

"On average, middle-aged consumers devote 39 percent of their retail expenditure to department store products and services, while for younger consumers the average is only 25 percent. Since the number of middle-aged people will increase dramatically within the next decade, department stores can expect retail sales to increase significantly during that period. Furthermore, to take advantage of the trend, these stores should begin to replace some of those products intended to attract the younger consumer with products intended to attract the middle-aged consumer."

Sample Prompt 3

The following appeared as part of an editorial in an industry newsletter:

"While trucking companies that deliver goods pay only a portion of highway maintenance costs and no property tax on the highways they use, railways spend billions per year maintaining and upgrading their facilities. The government should lower the railroad companies’ property taxes, since sending goods by rail is clearly a more appropriate mode of ground transportation than highway shipping. For one thing, trains consume only a third of the fuel a truck would use to carry the same load, making them a more cost-effective and environmentally sound mode of transport. Furthermore, since rail lines already exist, increases in rail traffic would not require building new lines at the expense of taxpaying citizens."

Sample Prompt 4

The following was excerpted from the speech of a spokesperson for Synthetic Farm Products, Inc.:

"Many farmers who invested in the equipment needed to make the switch from synthetic to organic fertilizers and pesticides feel that it would be too expensive to resume synthetic farming at this point. But studies of farmers who switched to organic farming last year indicate that their current crop yields are lower. Hence their purchase of organic farming equipment, a relatively minor investment compared to the losses that would result from continued lower crop yields, cannot justify persisting on an unwise course. And the choice to farm organically is financially unwise, given that it was motivated by environmental rather than economic concerns."

Sample Prompt 5

The following is part of a business plan created by the management of the Megamart grocery store:

"Our total sales have increased this year by 20 percent since we added a pharmacy section to our grocery store. Clearly, the customer’s main concern is the convenience afforded by one-stop shopping. The surest way to increase our profits over the next couple of years, therefore, is to add a clothing department along with an automotive supplies and repair shop. We should also plan to continue adding new departments and services, such as a restaurant and a garden shop, in subsequent years. Being the only store in the area that offers such a range of services will give us a competitive advantage over other local stores."

gmat example essay prompts

Homepage > GMAT Preparation > GMAT Essay – List of AWA Topics – 50 Practice Questions on GMAT AWA 2023

GMAT Essay – List of AWA Topics – 50 Practice Questions on GMAT AWA 2023

Posted by Suheb Hussain | Jan 25, 2023 | GMAT Focus Edition , GMAT IR and AWA , GMAT Preparation

GMAT Essay – List of AWA Topics – 50 Practice Questions on GMAT AWA 2023

This document contains practice questions that will help you improve the AWA section or the GMAT essay section. Discuss in your GMAT essay how well-reasoned you find the argument. Here are some ways you can do that while writing your GMAT essay for the AWA section. In your discussion, be sure to analyze the line of reasoning and the use of evidence in the argument. For example, you may need to consider what questionable assumptions underlie the thinking and what alternative explanations or counterexamples might weaken the conclusion. You can also discuss what sort of evidence would strengthen or refute the argument, what changes in the argument would make it more logically sound, and what, if anything, would help you better evaluate its conclusion.

We also recommend you read this article on how to score a perfect 6 on the GMAT AWA.

GMAT Essay AWA Practice Questions

Begin your GMAT preparation with the only prep company that has delivered more 700+ scores than any other GMAT club partner . Achieve GMAT 740+ with our AI driven tools that you personalized feedback at every step of your GMAT journey. Take our free trial today!

GMAT Essay practice question 1

  • The following appeared as part of an article in a trade magazine:

“During a recent trial period in which government inspections at selected meat-processing plants were more frequent, the number of bacteria in samples of processed chicken decreased by 50 percent on average from the previous year’s level. If the government were to institute more frequent inspections, the incidence of stomach and intestinal infections throughout the country could thus be cut in half. In the meantime, consumers of Excel Meats should be safe from infection because Excel’s main processing plant has shown more improvement in eliminating bacterial contamination than any other plant cited in the government report.”

Discuss how well reasoned . . . etc.

GMAT Essay practice question 2

2. The following appeared as part of an annual report sent to stockholders by Olympic Foods, a processor of frozen foods:

“Over time, the costs of processing go down because as organizations learn how to do things better, they become more efficient. In color film processing, for example, the cost of a 3-by-5-inch print fell from 50 cents for five-day service in 1970 to 20 cents for one-day service in 1984. The same principle applies to the processing of food. And since Olympic Foods will soon celebrate its 25th birthday, we can expect that our long experience will enable us to minimize costs and thus maximize profits.”

GMAT AWA practice question 3

3. The following appeared in a memorandum from the business department of the Apogee Company:

“When the Apogee Company had all its operations in one location, it was more profitable than it is today. Therefore, the Apogee Company should close down its field offices and conduct all its operations from a single location. Such centralization would improve profitability by cutting costs and helping the company maintain better supervision of all employees.”

If you are planning to take the GMAT, we can help you with a personalized study plan and give you access to quality online content to prepare. Write to us at  [email protected] . We are the  most reviewed GMAT prep company on gmatclub with more than 2500 reviews. Why don’t you take a free trial and judge for yourself?

GMAT Essay practice question 4

4. The following appeared in a memorandum issued by a large city’s council on the arts:

“In a recent citywide poll, 15 percent more residents said that they watch television programs about the visual arts than was the case in a poll conducted five years ago. During these past five years, the number of people visiting our city’s art museums has increased by a similar percentage. Since the corporate funding that supports public television, where most of the visual arts programs appear, is now being threatened with severe cuts, we can expect that attendance at our city’s art museums will also start to decrease. Thus some of the city’s funds for supporting the arts should be reallocated to public television.”

GMAT Essay practice question 5

5. The following appeared in an announcement issued by the publisher of The Mercury , a weekly newspaper:

“Since a competing lower-priced newspaper, The Bugle , was started five years ago, The Mercury ’s circulation has declined by 10,000 readers. The best way to get more people to read The Mercury is to reduce its price below that of The Bugle , at least until circulation increases to former levels. The increased circulation of The Mercury will attract more businesses to buy advertising space in the paper.”

Attention test takers! GMAC has recently announced a new version of the GMAT test called the GMAT Focus edition. This new test will be launched in the latter part of 2023, while the current version of the test will be available till at least early 2024.

Read this article to know the key differences between the current version and the GMAT focus edition .

Also, watch the following video to learn all about the GMAT Focus edition and also understand how to decide which version of the exam to take- The current version or the GMAT Focus edition.

GMAT Essay practice question 6

6.  The following appeared in a report presented for discussion at a meeting of the directors of a company that manufactures parts for heavy machinery:

“The falling revenues that the company is experiencing coincide with delays in manufacturing. These delays, in turn, are due in large part to poor planning in purchasing metals. Consider further that the manager of the department that handles purchasing of raw materials has an excellent background in general business, psychology, and sociology, but knows little about the properties of metals. The company should, therefore, move the purchasing manager to the sales department and bring in a scientist from the research division to be manager of the purchasing department.”

GMAT Essay practice question 7

7. The following appeared in a research paper written for an introductory economics course:

“For the past century, an increase in the number of residential building permits issued per month in a particular region has been a reliable indicator of coming improvements to that region’s economy. If the monthly number of residential building permits issued rises consistently for a few months, the local unemployment rate almost always falls and economic production increases. This well-established connection reveals an effective method by which a regional government can end a local economic downturn: relax regulations governing all construction so that many more building permits can be issued.”

Discuss how well reasoned . . .etc.

GMAT Essay practice question 8

8.  The following appeared in a memorandum to the work-group supervisors of the GBS Company:

“The CoffeeCart beverage and food service located in the lobby of our main office building is not earning enough in sales to cover its costs, and so the cart may discontinue operating at GBS. Given the low staff morale, as evidenced by the increase in the number of employees leaving the company, the loss of this service could present a problem, especially since the staff morale questionnaire showed widespread dissatisfaction with the snack machines.

Therefore, supervisors should remind the employees in their group to patronize the cart—after all, it was leased for their convenience so that they would not have to walk over to the cafeteria on breaks.”

GMAT Essay practice question 9

9. The following appeared in a memorandum written by the chair of the music department to the president of Omega University:

“Mental health experts have observed that symptoms of mental illness are less pronounced in many patients after group music-therapy sessions, and job openings in the music-therapy field have increased during the past year. Consequently, graduates from our degree program for music therapists should have no trouble finding good positions. To help improve the financial status of Omega University, we should, therefore, expand our music-therapy degree program by increasing its enrollment targets.”

GMAT Essay practice question 10

10. The following appeared in a memorandum from the vice president of Gigantis, a development company that builds and leases retail store facilities:

“Nationwide over the past five years, sales have increased significantly at outlet stores that deal exclusively in reduced-price merchandise. Therefore, we should publicize the new mall that we are building at Pleasantville as a central location for outlet shopping and rent storage space only to outlet companies. By taking advantage of the success of outlet stores, this plan should help ensure full occupancy of the mall and enable us to recover quickly the costs of building the mall.”

11. The following appeared in a memorandum from the business planning department of Avia Airlines:

“Of all the cities in their region, Beaumont and Fletcher are showing the fastest growth in the number of new businesses. Therefore, Avia should establish a commuter route between them as a means of countering recent losses on its main passenger routes. And to make the commuter route more profitable from the outset, Avia should offer a 1/3 discount on tickets purchased within two days of the flight. Unlike tickets bought earlier, discount tickets will be nonrefundable, and so gain from their sale will be greater.”

12. The following appeared in a speech by a stockholder of Consolidated Industries at the company’s annual stockholders’ meeting:

“In the computer hardware division last year, profits fell significantly below projections, the product line decreased from 20 to only 5 items, and expenditures for employee benefits increased by 15 percent. Nevertheless, Consolidated’s board of directors has approved an annual salary of more than $1 million for our company’s chief executive officer. The present board members should be replaced because they are unconcerned about the increasing costs of employee benefits and salaries, in spite of the company’s problems generating income.”

13.  The following appeared as part of a business plan by the Capital Idea investment firm:

“In recent years the worldwide demand for fish has grown, and improvements in fishing technology have made larger catches and thus increased supply possible: for example, last year’s tuna catch was 9 percent greater than the previous year’s. To capitalize on these trends, we should, therefore, invest in the new tartfish processing plant on Tartfish Island, where increasing revenues from tourism indicate a strong local economy.”

14. The following appeared as part of an article in a weekly newsmagazine:

“The country of Oleum can best solve the problem of its balance of trade deficit by further increasing the tax on its major import, crude oil. After Oleum increased the tax on imported crude oil four months ago, consumption of gasoline declined by 20 percent. Therefore, by imposing a second and significantly higher tax increase next year, Oleum will dramatically decrease its balance of trade deficit.”

15. The following appeared in a memorandum from the human resources department of HomeStyle, a house remodeling business:

“This year, despite HomeStyle’s move to new office space, we have seen a decline in both company morale and productivity, and a corresponding increase in administrative costs. To rectify these problems, we should begin using a newly developed software package for performance appraisal and feedback. Managers will save time by simply choosing comments from a preexisting list; then the software will automatically generate feedback for the employee. The human resources department at CounterBalance, the manufacturer of the countertops we install, reports satisfaction with the package.”

16. The following appeared in a memorandum written by the managing director of the Exeunt Theater Company:

“Now that we have moved to a larger theater, we can expect to increase our revenues from ticket sales. To further increase profits, we should start producing the plays that have been most successful when they were performed in our nation’s largest cities. In addition, we should hire the Adlib Theater Company’s director of fund-raising, since corporate contributions to Adlib have increased significantly over the three years that she has worked for Adlib.”

Do you also want a 700+ on the GMAT? Try out our Free Trial today and get access to 10+ hours of free interactive video lessons, 400+ GMAT-like questions, and AI-drive tools that help you track your progress. We are the most reviewed GMAT Preparation company on the GMAT Club , with 2500+ reviews!

17. The following appeared in a memorandum from a regional supervisor of post office operations:

“During a two-week study of postal operations, the Presto City post office handled about twice as many items as the Lento City post office, even though the cities are about the same size. Moreover, customer satisfaction appears to be higher in Presto City, since the study found fewer complaints regarding the Presto City post office. Therefore, the postmasters at these two offices should exchange assignments: the Presto City postmaster will solve the problems of inefficiency and customer dissatisfaction at the Lento City office while the Lento City postmaster learns firsthand the superior methods of Presto City.”

18. The following appeared in a memorandum from the human resources department of Diversified Manufacturing:

“Managers at our central office report that their employees tend to be most productive in the days immediately preceding a vacation. To help counteract our declining market share, we could increase the productivity of our professional staff members, who currently receive four weeks paid vacation a year, by limiting them to a maximum of one week’s continuous vacation time. They will thus take more vacation breaks during a year and give us more days of maximum productivity.”

19. The following appeared in a memorandum from the marketing department of a children’s clothing manufacturer:

“Our HuggyBunny brand is the bestselling brand of children’s clothing. Parents everywhere recognize the HuggyBunny logo as a mark of quality, and most of our customers show great brand loyalty. Sales reports have shown that parents are more likely to buy children’s clothes with the familiar HuggyBunny brand and logo than otherwise identical clothes without it. Therefore, if we use the HuggyBunny brand name and logo for the new line of clothing for teenagers that our company will soon be introducing, that clothing will sell better than it would if we labeled it with a new brand name and logo.”

20. The following appeared in a memorandum from the president of Aurora, a company that sells organic milk (milk produced without the use of chemical additives):

“Sales of organic food products in this country have tripled over the past five years. If Aurora is to profit from this continuing trend, we must diversify and start selling products such as organic orange juice and organic eggs in addition to our regular product line. With the recent increase of articles in health magazines questioning the safety of milk and other food products, customers are even more likely to buy our line of organic products. And to help ensure our successful expansion, we should hire the founder of a chain of health-food stores to serve as our vice president of marketing.”

21. The following appeared as part of an article in a newsletter for farmers:

“Users of Solacium, a medicinal herb now grown mainly in Asia, report that it relieves tension and promotes deep sleep. A recent study indicates that a large number of college students who took pills containing one of the ingredients in Solacium suffered less anxiety. To satisfy the anticipated demands for this very promising therapeutic herb and to reap the financial benefits, farmers in this country should begin growing it.”

22. The following appeared as part of the business plan of the Capital Idea investment firm:

“Across town in the Park Hill district, the Thespian Theater, Pizzazz Pizza, and the Niblick Golf Club have all had business increases over the past two years. Capital Idea should, therefore, invest in the Roxy Playhouse, the Slice-o’- Pizza, and the Divot Golf Club, three new businesses in the Irongate district. As a condition, we should require them to participate in a special program: Any customer who patronizes two of the businesses will receive a substantial discount at the third. By motivating customers to patronize all three, we will thus contribute to the profitability of each and maximize our return.”

23. The following appeared in a memorandum from the owner of Carlo’s Clothing to the staff:

“Since Disc Depot, the music store on the next block began a new radio advertising campaign last year, its business has grown dramatically, as evidenced by the large increase in foot traffic into the store. While the Disc Depot’s owners have apparently become wealthy enough to retire, profits at Carlo’s Clothing have remained stagnant for the past three years. In order to boost our sales and profits, we should, therefore, switch from newspaper advertising to frequent radio advertisements like those for Disc Depot.”

24. The following appeared in a letter to the editor of a River City newspaper:

“The Clio Development Group’s plan for a multilevel parking garage on Dock Street should be approved in order to strengthen the economy of the surrounding area. Although most of the buildings on the block would have to be demolished, they are among the oldest in the city and thus of little current economic value. Those who oppose the project should realize that historic preservation cannot be the only consideration: even Athens or Jerusalem will knock down old buildings to put up new ones that improve the local economy.”

25. The following appeared in a memorandum from the publisher to the staff of The Clarion , a large metropolitan newspaper:

“During the recent campaign for mayor, a clear majority of city readers who responded to our survey indicated a desire for more news about city government. To increase circulation, and thus our profits, we should therefore consistently devote a greater proportion of space in all editions of The Clarion to coverage of local news.”

26. The following appeared in a corporate planning memorandum for a company that develops amusement parks:

“Because travel from our country to foreign countries has increased dramatically in recent years, our next project should be a ‘World Tour’ theme park with replicas of famous foreign buildings, rides that have international themes, and refreshment stands to serve only foods from the country represented by the nearest ride. The best location would be near our capital city, which has large percentages of international residents and of children under the age of 16. Given the advantages of this site and the growing interest in foreign countries, the ‘World Tour’ theme park should be as successful as our space-travel theme park, where attendance has increased tenfold over the past decade.”

27.  The following appeared in a letter to the editor of a River City newspaper:

“The Clio Development Group should not be permitted to build a multilevel parking garage on Dock Street since most of the buildings on the block would have to be demolished. Because these buildings were erected decades ago, they have historic significance and must, therefore, be preserved as economic assets in the effort to revitalize a restored riverfront area. Recall how Lakesburg has benefited from business increases in its historic downtown center. Moreover, there is plenty of vacant land for a parking lot elsewhere in River City.”

28. The following appeared in a letter to the editor of a local newspaper:

“Our city council’s neglect of the impoverished Railroad Flats neighborhood has left businesses with little incentive to locate there. Building a new professional football stadium in the neighborhood would solve this problem. Thousands of football fans would travel to the area to see games, and they would buy from local merchants, encouraging new businesses to open. So our city council should move quickly to fund the construction of a professional football stadium in Railroad Flats in order to help the neighborhood develop a thriving economy.”

29. The following appeared in a memorandum from the director of marketing for a pharmaceutical company:

“According to a survey of 5,000 urban residents, the prevalence of stress headaches increases with educational level, so that stress headaches occur most often among people with graduate-school degrees. It is well established that, nationally, higher educational levels usually correspond with higher levels of income. Therefore, in marketing our new pain remedy, Omnilixir, we should send free samples primarily to graduate students and to people with graduate degrees, and we should concentrate on advertising in professional journals rather than in general interest magazines.”

30. The following appeared as part of an editorial in the Waymarsh city newspaper:

“Last year the parents of first graders in our school district expressed satisfaction with the reading skills their children developed but complained strongly about their children’s math skills. To remedy this serious problem and improve our district’s elementary education, everyone in the teacher-training program at Waymarsh University should be required to take more courses in mathematics.”

31. The following appeared in a memorandum from the business office of the Lovin’ Cupful, a national restaurant chain:

“The Lovin’ Cupful franchises in our northeast region have begun serving customers Almost, a brand new powdered instant tea, in place of brewed tea. Waiters report that only about 2 percent of the customers have complained and that customers who want refills typically ask for ‘more tea.’ It appears, then, that 98 percent of the customers are perfectly happy with the switch, or else they cannot tell powdered instant from brewed tea. Therefore, in order to take advantage of the lower price per pound of Almost, all of our restaurants should begin substituting it for brewed tea.”

32. The following appeared as a memorandum from the vice-president of the Dolci candy company:

“Given the success of our premium and most expensive line of chocolate candies in a recent taste test and the consequent increase in sales, we should shift our business focus to producing additional lines of premium candy rather than our lower-priced, ordinary candies. When the current economic boom ends and consumers can no longer buy major luxury items, such as cars, they will still want to indulge in small luxuries, such as expensive candies.”

33. The following appeared in a memorandum from the director of research and development at Ready-to-Ware, a software engineering firm:

“The package of benefits and incentives that Ready-to-Ware offers to professional staff is too costly. Our quarterly profits have declined since the package was introduced two years ago, at the time of our incorporation. Moreover, the package had little positive effect, as we have had only marginal success in recruiting and training high-quality professional staff. To become more profitable again, Ready-to-Ware should, therefore, offer the reduced benefits package that was in place two years ago and use the savings to fund our current research and development initiatives.”

34. The following appeared in a memorandum from the assistant manager of Pageturner Books:

“Over the past two years, Pageturner’s profits have decreased by 5 percent, even though we have added a popular café as well as a music section selling CDs and tapes. At the same time, we have experienced an increase in the theft of merchandise. We should, therefore, follow the example of Thoreau Books, which increased its profits after putting copies of its most frequently stolen books on a high shelf behind the payment counter. By doing likewise with copies of the titles that our staff reported stolen last year, we too can increase profitability.”

35. The following appeared in a memorandum to a team developing accounting software for SmartPro Software, Inc.:

“Currently, more professional accountants use SmartPro accounting software than any other brand. However, in the market for personal accounting software for non-professionals to use in preparing their income tax returns, many of our competitors are outselling us. In surveys, our professional customers repeatedly say that they have chosen SmartPro Software because our most sophisticated software products include more advanced special features than competing brands. Therefore, the most effective way for us to increase sales of our personal accounting software for home users would clearly be to add the advanced special features that our professional software products currently offer.”

36. The following appeared in a memorandum written by the assistant manager of a store that sells gourmet food items from various countries:

“A local wine store made an interesting discovery last month: it sold more French than Italian wine on days when it played recordings of French accordion music, but it sold more Italian than French wine on days when Italian songs were played. Therefore, I recommend that we put food specialties from one particular country on sale for a week at a time and play only music from that country while the sale is going on. By this means we will increase our profits in the same way that the wine store did, and we will be able to predict more precisely what items we should stock at any given time.”

37. The following appeared in a memo to the Saluda town council from the town’s business manager:

“Research indicates that those who exercise regularly are hospitalized less than half as often as those who don’t exercise. By providing a well-equipped gym for Saluda’s municipal employees, we should be able to reduce the cost of our group health insurance coverage by approximately 50 percent and thereby achieve a balanced town budget.”

38. The following appeared as part of a letter to the editor of a local newspaper:

“Bayview High School is considering whether to require all of its students to wear uniforms while at school. Students attending Acorn Valley Academy, a private school in town, earn higher grades on average and are more likely to go on to college. Moreover, Acorn Valley reports few instances of tardiness, absenteeism, or discipline problems. Since Acorn Valley requires its students to wear uniforms, Bayview High School would do well to follow suit and require its students to wear uniforms as well.”

39. The following appeared as part of a memorandum from the loan department of the Frostbite National Bank:

“We should not approve the business loan application of the local group that wants to open a franchise outlet for the Kool Kone chain of ice cream parlors. Frostbite is known for its cold winters, and cold weather can mean slow ice cream sales. For example, even though Frostbite is a town of 10,000 people, it has only one ice cream spot—the Frigid Cow. Despite the lack of competition, the Frigid Cow’s net revenues fell by 10 percent last winter.”

40. The following appeared in a letter from a staff member in the office of admissions at Argent University:

“The most recent nationwide surveys show that undergraduates choose their major field primarily based on their perception of job prospects in that field. At our university, economics is now the most popular major, so students must perceive this field as having the best job prospects. Therefore, we can increase our enrollment if we focus our advertising and recruiting on publicizing the accomplishments of our best-known economics professors and the success of our economics graduates in finding employment.”

41. The following appeared as part of a business plan created by the management of the Take Heart Fitness Center:

“After opening the new swimming pool early last summer, Take Heart saw a 12 percent increase in the use of the center by its members. Therefore, in order to increase membership in Take Heart, we should continue to add new recreational facilities in subsequent years: for example, a multipurpose game room, a tennis court, and a miniature golf course. Being the only center in the area offering this range of activities would give us a competitive advantage in the health and recreation market.”

42. The following appeared as part of an article in the book section of a newspaper:

“Currently more and more books are becoming available in electronic form—either free-of-charge on the Internet or for a very low price-per-book on compact disc.* Thus literary classics are likely to be read more widely than ever before. People who couldn’t have purchased these works at bookstore prices will now be able to read them for little or no money; similarly, people who find it inconvenient to visit libraries and wait for books to be returned by other patrons will now have access to whatever classic they choose from their home or work computers. This increase in access to literary classics will radically affect the public taste in reading, creating a far more sophisticated and learned reading audience than has ever existed before.”

*A compact disc is a small portable disc capable of storing relatively large amounts of data that can be read by a computer.

43. The following appeared as an editorial in a magazine concerned with educational issues:

“In our country, the real earnings of men who have only a high-school degree have decreased significantly over the past 15 years, but those of male college graduates have remained about the same. Therefore, the key to improving the earnings of the next generation of workers is to send all students to college. Our country’s most important educational goal, then, should be to establish enough colleges and universities to accommodate all high school graduates.”

44. The following appeared in an editorial from a newspaper serving the town of Saluda:

“The Saluda Consolidated High School offers more than 200 different courses from which its students can choose. A much smaller private school down the street offers a basic curriculum of only 80 different courses, but it consistently sends a higher proportion of its graduating seniors on to college than Consolidated does. By eliminating at least half of the courses offered there and focusing on a basic curriculum, we could improve student performance at Consolidated and also save many tax dollars.”

45. The following appeared as part of an article in a trade magazine for breweries:

“Magic Hat Brewery recently released the results of a survey of visitors to its tasting room last year. Magic Hat reports that the majority of visitors asked to taste its low-calorie beers. To boost sales, other small breweries should brew low-calorie beers as well.”

46. The following appeared in a memorandum sent by a vice-president of the Nadir Company to the company’s human resources department:

“Nadir does not need to adopt the costly ‘family-friendly’ programs that have been proposed, such as part-time work, work at home, and job-sharing. When these programs were made available at the Summit Company, the leader in its industry, only a small percentage of employees participated in them. Rather than adversely affecting our profitability by offering these programs, we should concentrate on offering extensive training that will enable employees to increase their productivity.”

47. The following appeared in a letter to prospective students from the admissions office at Plateau College:

“Every person who earned an advanced degree in science or engineering from Olympus University last year received numerous offers of excellent jobs. Typically, many graduates of Plateau College have gone on to pursue advanced degrees at Olympus. Therefore, enrolling as an undergraduate at Plateau College is a wise choice for students who wish to ensure success in their careers.”

48. The following was excerpted from an article in a farming trade publication:

“Farmers who switched from synthetic to organic farming last year have seen their crop yields decline. Many of these farmers feel that it would be too expensive to resume synthetic farming at this point, given the money that they invested in organic farming supplies and equipment. But their investments will be relatively minor compared to the losses from continued lower crop yields. Organic farmers should switch to synthetic farming rather than persist in an unwise course. And the choice to farm organically is financially unwise, given that it was motivated by environmental rather than economic concerns.”

49. The following appeared as part of an article in a computer magazine:

“A year ago Apex Manufacturing bought its managers computers for their homes and paid for telephone connections so that they could access Apex computers and data files from home after normal business hours. Since last year, productivity at Apex has increased by 15 percent. Other companies can learn from the success at Apex: given home computers and access to company resources, employees will work additional hours at home and thereby increase company profits.”

50. The following appeared in the editorial section of a local paper:

“Applications for advertising spots on KMTV, our local cable television channel, decreased last year. Meanwhile, a neighboring town’s local channel, KOOP, changed its focus to farming issues and reported an increase in advertising applications for the year. To increase applications for its advertisement spots, KMTV should focus its programming on farming issues as well.”

About The Author

gmat example essay prompts

Suheb Hussain

Don't limit yourself, don't miss these.

gmat example essay prompts

Helpful posts Curated just for you!

gmat example essay prompts

You might also like

gmat example essay prompts

Browse Related Topics

gmat example essay prompts

400+ Practice questions with detailed solutions

10+ hours of ai-driven video lessons, adaptive mock test with esr+ analysis.

FR Île-de-France

Recently viewed courses

Recently viewed.

Find Your Dream School

This site uses various technologies, as described in our Privacy Policy, for personalization, measuring website use/performance, and targeted advertising, which may include storing and sharing information about your site visit with third parties. By continuing to use this website you consent to our Privacy Policy and Terms of Use .

   COVID-19 Update: To help students through this crisis, The Princeton Review will continue our "Enroll with Confidence" refund policies. For full details, please click here.

GMAT Sample Questions

Hidden laptop with hands typing, displaying a test diagram above the keyboard

Want a preview of the question types you'll face on the GMAT? Try your hand at the GMAT practice questions below. Then, check your answers against our in-depth explanations to see how you did.

We pulled these GMAT sample questions from our book Cracking the GMAT and from our test prep course materials. For more GMAT practice, take a full-length practice test with us held under the same testing conditions as the real thing. Find out how you'd score, and get  a personalized score report from us that shows your strengths and weaknesses.

  • GMAT Verbal Questions 
  • GMAT Math Questions
  • GMAT Integrated Reasoning Questions 
  • Essay Prompt 

Below you'll find sample GMAT questions covering the three question types you'll encounter on the Verbal section: Sentence Correction , Critical Reasoning, and Reading Comprehension.

GMAT Sentence Correction Questions

1. In order to better differentiate its product from generic brands, the cereal company first hired a marketing firm that specializes in creating campaigns to build brand awareness and then retools its factory to produce a variety of different shapes of cereal. (A) then retools its factory to produce a variety of different shapes of cereal (B) retools its factory to produce a variety of different shapes of cereal (C) then retooled its factory to produce a variety of different shapes of cereal (D) then will retool its factory to produce a variety of different shapes of cereal (E) then produces a variety of different shapes of cereal through retooling its factory

Answer: (C) The actions of the cereal company are not in parallel form. First the company hired then it retools . Eliminate choice (A). Choice (B) still has the same error. Choice (D) changes the verb form incorrectly to the future tense. Choice (E) rewrites the sentence but retains the error.

[+] See the Answer

2. Believed to be one of the first widely read female authors of the Western world, Christine de Pizan's masterwork The Book of the City of the Ladies , was written in 1405 and is a history of the Western world from the woman's point-of-view. (A) Believed to be one of the first widely read female authors of the Western world (B) Written by one of the first widely read female authors of the Western world (C) One of the first widely read female authors of the Western world, as some believe (D) Written by what some believe as one of the first widely read female authors of the Western world (E) Believed by some as one of the first works by a widely read female author in the Western world

Answer: (B) As written, this sentence has a misplaced modifier error: the book, The Book of the City of the Ladies isn't believed by anyone to be an author— Christine de Pizan is. Choices (A) and (C) repeat that error and can be eliminated. Choices (B) and (D) both change the introductory phrase to clearly refer to a written work, but choice (D) uses the incorrect idiom believe as instead of the correct form, believe to be . Choice (E) repeats that idiom error.

GMAT Critical Reasoning Questions

1. One food writer wrote that reducing the amount of animal products in one's diet can contribute to better health and well-being. Based on this claim, some people are completely eliminating meat from their diets in order to be healthier. The argument above relies on which of the following assumptions?

Answer: (B) The argument states that some people are eliminating meat from their diets because reducing the amount of animal products in one's diet can lead to better health. Meat is only one type of animal product, however. The argument assumes that by eliminating meat, the people are reducing the total amount of animal products in their diets. Choice (A) addresses increasing the amount of vegetables and grains, but the argument just deals with animal products. Choice (B) correctly addresses the people who are eliminating meat and states that those people are not increasing their consumption of dairy, which is another instance of using animal products. Thus, these people are actually reducing the amount of animal products in their diets. Choice (C) addresses most food writers, who are irrelevant to this argument. Choice (D) addresses health lifestyles, which are irrelevant to this particular argument. Choice (E) addresses the reasons behind not eating animal products, which is irrelevant to the argument.

2. Studies reveal that a daily exercise regimen helps stroke survivors regain dexterity in their extremities. Being given an exercise routine and having a consultation with a doctor about the exercise routine have been shown to be effective mechanisms to get patients to exercise daily. From the above information, which of the following statements can be reasonably inferred? (A) A stroke survivor that is given a detailed exercise plan and consults her physician about the plan will regain full dexterity in her extremities. (B) If a stroke survivor is not given an exercise plan and does not consult with a doctor, she will not regain dexterity in her extremities. (C) Stroke survivors who are given an exercise routine and consult with a doctor about that routine will sometimes regain dexterity in their extremities. (D) Being given an exercise routine and having a consultation with a doctor about the routine is the best way to help a stroke survivor regain dexterity in their extremities. (E) Only being given an exercise routine is necessary to regenerate dexterity in the extremities of seniors who have suffered a stroke.

Answer: (C) This is an inference question, so evaluate the passage and then look for an answer choice that can be reasonably inferred from the information. The passage states that a daily exercise regimen helps stroke survivors regain dexterity in their extremities and that survivors who are given an exercise routine and who have a consultation with a doctor about the routine have been shown to be effective at getting patients to exercise daily . So it can be inferred that if a survivor is given a routine and consults with a doctor, they are more likely to exercise daily, which will help them regain dexterity. Choice (A) is an example of extreme language. The phrasing will regain full dexterity is not promised in the information in the passage, as the passage only states that a routine and consultations may help a survivor exercise more. Eliminate (A). Choice (B) is also an example of extreme language. There is no way to discern from the information provided that a strong survivor would not regain dexterity without an exercise routine and a consultation, so eliminate (B). Choice (C) is a reasonable inference to make from the information in the passage so keep (C). Choice (D) also contains the extreme language best way . The information does not compare this method with any other method so eliminate (D). Choice (E) is recycled language and does not address consulting with a doctor so eliminate (E). The correct answer is (C).

GMAT Reading Comprehension Questions

Although oft-maligned in modern culture, the pigeon once stood not only for speed and reliability but also for grace and beauty. Darwin himself became a pigeon fancier after beginning to work with the humble Columbia livia , discovering them to be more fascinating than he had formerly believed. During the Victorian age, in fact, raising show pigeons was a popular hobby, with new breeds continuously arising as amateur (and not-so-amateur) ornithologists crossed animals in the hopes of creating ever more fantastic creatures. One of the most sought-after varieties was known as the Almond Tumbler, a name presumably derived from the color of the birds combined with the distinctive flight style. Over the course of many generations, this bird was so manipulated as to have a beak so small as to prevent the adult birds from feeding their offspring. And yet, it was wildly popular, drawing high prices at auctions and high prizes at competitions. How then did an animal once so well-loved come to be so loathed? As recently as World War II, the military used pigeons to carry messages but today, many people would kick a pigeon before they would feed one. Perhaps it is just a problem of population density - a lack of esteem for that which is ubiquitous. Pigeons have become our constant urban companions and, as such, have been transformed from symbols of peace, plenty, and prosperity, to representatives of disease and decay.

1. The primary purpose of this passage is to (A) convince the reader of the nobility of the pigeon, based on its history as a symbol of virtue (B) dissuade the reader from mistreating a once-majestic animal that has fallen from favor (C) rebut claims that the pigeon carries disease any more frequently than do other domestic animals (D) promote a renewal of pigeon fancying and a resurgence of breeds such as the Almond Tumbler (E) suggest that there might be more to the story of some urban wildlife than is commonly known

Answer: (E) The passage gives a brief description of the pigeon's place in recent human history and then goes on to contrast that with modern perspectives of the birds. Choice (A) goes too far—the author doesn't give any indication of believing the pigeon to be noble. Choice (B) focuses too specifically on a side comment in the second paragraph. Choice (C) also focuses too specifically on a side comments—the passage is not primarily about disease. Choice (D) is too strong—the passage isn't really promoting any specific action. Choice (E) remains neutral and informational, as does the passage.

2. The case of the Almond Tumbler is most analogous to which of the following? (A) a strain of wheat that can be grown in plentiful quantities but loses much of its nutritional value in the process (B) Arabian horses that are able to run at phenomenal speeds due to centuries of careful breeding designed to enhance those physical attributes (C) vitamins that were purported to provide all of the necessary nutrients but have since been found not to be very effective (D) the dachshund, a popular breed of dog that is nonetheless prone to severe back problems, due to weaknesses exacerbated by targeted breeding (E) the wild rock doves that are most commonly found nesting in the faces of cliffs far from human habitation

Answer: (D) The Almond Tumbler is described as a breed of pigeon that was very popular during the Victorian era. The passage also mentions that the selective breeding used to create that particular kind of bird also led to tiny beaks that kept parent birds from feeding their babies. Therefore, the best analogy would be another animal that is popular even though it has problems due to its design. Choice (A) is incorrect because it leaves out the aspect of popularity. Choice (B) is only positive and you need something that's also negative. Choice (C) is not about something that has been bred for a specific purpose, nor does it deal with popularity. Choice (D) correctly refers to a popular animal with a common health problem. Choice (E) does not refer to pigeons that have been bred by humans.

3. The passage suggests that (A) pigeons were once known for flying with celerity (B) the Almond Tumbler was the most beautiful breed of pigeon (C) Darwin was infatuated with his fancy pigeons (D) modern pigeons are dirtier than the fancy pigeons of yore (E) only scientists should breed new kinds of animals

Answer: (A) For a question this open-ended, it's usually best to check each of the answers against the passage. Choice (A) appears to match the opening line of the passage, which states that the pigeon once stood not only for speed and reliability. Choice (B) goes too far—although many Victorians seems to have loved the Tumbler, there's no evidence that it was definitively the most beautiful. Choice (C) also goes too far—the passage mentions that Darwin was fascinated by his pigeons, not that he was infatuated. Choice (D) draws an incorrect assumption—the passage comments that the common opinion has changed, not the pigeon itself. Choice (E) is not supported by the passage, which states that amateurs, as well as trained individuals, bred pigeons.

Below you'll find GMAT sample questions covering the two question types you'll encounter on the Quantitative section: Problem Solving and Data Sufficiency.

Problem Solving Questions

1. A certain company sells tea in loose leaf and bagged form, and in five flavors: Darjeeling, earl grey, chamomile, peppermint, and orange pekoe. The company packages the tea in boxes that contain either 8 ounces of tea of the same flavor and the same form, or 8 ounces of tea of 4 different flavors and the same form. If the order in which the flavors are packed does not matter, how many different types of packages are possible? (A) 12 (B) 15 (C) 20 (D) 25 (E) 30

Answer: (C) Begin by figuring out how many different ways you can package the tea in boxes that contains 8 ounces of tea, all of the same flavor. There are five flavors, each flavor can come in either loose leaf or bagged form, so 5 flavors x 2 forms = 10 different ways to package the tea in boxes that contain only one flavor each. Now find the number of different ways to package 4 different flavors of the same form per box. In this case, you must choose 4 of 5 possible flavors, and order does not matter, so the formula is 5 x 4 x 3 x 2 ⁄ 4 x 3 x 2 x 1 = 5 different ways to combine the 4 flavors. Each combination can come in either loose leaf for bagged form, so you have 2 different forms x 5 different combinations = 10 total possible ways to combine the 4 flavors in either bagged or loose-leaf form. Thus, the total number of combinations is 10 + 10 = 20 total combinations. The answer is choice (C).

2. Karen sold her house at a loss of 25 percent of the price that she originally paid for the house, and then bought another house at a price of 30 percent less than the price she originally paid for her first house. If she sold the first house for $225,000, what was her net gain, in dollars, for the two transactions? (A) $15,000 (B) $25,000 (C) $60,000 (D) $75,000 (E) $90,000

Answer: (A) If Karen sold her first house for $225,000 and at a loss of 25 percent, then 25 percent of the original price equals $225,000. 75 ⁄ 100 x = 225,000, so x, or the price she originally paid, equals $300,000. Thus, Karen lost $75,000 on the sale of her first house. If she bought a second house for a price of 30 percent less than $300,000, then the second house cost $210,000, so she gained $90,0000. $90,000 - $75,000 = $15,000, so the answer is choice (A).

Sample Data Sufficiency Questions

1. In a certain company, at least 200 people own manual transmission vehicles. If 12 percent of the people who own manual transmission vehicles also own automatic transmission vehicles, do more people own automatic transmission vehicles than own manual transmission vehicles? (1) 5 percent of the people who own an automatic transmissions vehicle also own a manual transmission vehicle. (2) 15 people own both an automatic transmission vehicle and a manual transmission vehicle. (A) Statement (1) ALONE is sufficient, but statement (2) alone is not sufficient. (B) Statement (2) ALONE is sufficient, but statement (1) alone is not sufficient. (C) BOTH statements TOGETHER are sufficient, but NEITHER statement ALONE is sufficient. (D) EACH Statement ALONE is sufficient. (E) Statements (1) and (2) TOGETHER are NOT sufficient to answer the question asked, and additional data are needed.

Answer: (A) According to statement (1), 5 percent of the people who own an automatic transmission vehicle also own a manual transmission vehicle. The question also indicates that 12 percent of the people who own a manual transmission vehicle also own an automatic transmission vehicle. Both figures relate to the total number who own both, so that means that 5 percent of the automatic transmission owners = 12 percent of the manual transmission owners. The overlap in ownership makes up a smaller percent of those who own automatic transmission vehicles, so there must be more people who own automatic transmission vehicles. Statement (1) is sufficient, so you can eliminate choices (B), (C), and (E). Statement (2) indicates that 15 people own both an automatic transmission vehicle and a manual transmission vehicle, so you know that 12 percent of the people who own a manual transmission is equal to 15 people. 12 ⁄ 100 = 15, so x = 125. Thus, there are 125 people who own a manual transmission vehicle. However, you have no further information to allow you to calculate the number of people who own automatic transmission vehicles, so statement (2) is insufficient. The answer is choice (A).

2. What is the value of x ⁄ 2 ? (1) x is 1 ⁄ 5 less than 9 ⁄ 10 (2) x is between 2 ⁄ 5 and 4 ⁄ 5 (A) Statement (1) ALONE is sufficient, but statement (2) alone is not sufficient. (B) Statement (2) ALONE is sufficient, but statement (1) alone is not sufficient. (C) BOTH statements TOGETHER are sufficient, but NEITHER statement ALONE is sufficient. (D) EACH Statement ALONE is sufficient. (E) Statements (1) and (2) TOGETHER are NOT sufficient to answer the question asked, and additional data are needed.

Answer: (A) Statement (1) allows you to find the value of x, so you can answer the question. (If x is 1 ⁄ 5 less than 9 ⁄ 10 , then 9 ⁄ 10 - 1 ⁄ 5 = x. 1 ⁄ 5 = 2 ⁄ 10 , so x equals 9 ⁄ 10 - 2 ⁄ 10 = 7 ⁄ 10 . If x equals 7 ⁄ 10 , then x ⁄ 2 = 7 ⁄ 10 divided by 2, or 7 ⁄ 20 .) Statement (1) is sufficient, so eliminate choices (B), (C), and (E). According to statement (2), x is between 2 ⁄ 5 and 4 ⁄ 5 . That means that one possible value for x is 3 ⁄ 5 , but another possible value is 7 ⁄ 10 . Statement (2) is insufficient, so the answer is choice (A).

Below you'll find examples of how you'll be asked to use a chart, graph, or table to answer questions on the Integrated Reasoning section.

Sample Integrated Reasoning Questions

Item 1: Andre is buying gifts for his office staff. He wants to spend exactly $280 and he can buy either sweatshirts, which cost $22, or baseball caps, which cost $26. In the table below, choose the number of sweatshirts and the number of baseball caps that Andre should buy.

Answer: Sweatshirts, 8; Baseball caps 4 To solve this question, systematically test out the answer choices. The equation you need to solve is 22s + 26h = 280, in which both s and h are integers and s represents the number of sweatshirts and h represents the number of baseball caps. So, start with plugging in 4 for sweatshirts and see if the number of baseball caps is an integer. 22(4) +26h = 280 h = 7.38 Since the number of baseball caps is not an integer, Andre could not have bought 4 sweatshirts. Keep trying more sweatshirts one by one until you find an answer that will you an integer value for baseball caps. 8 sweatshirts will give you 4 baseball caps.

GMAT sample question

Question 2-1 The ratio of the U.S. population in 2000 to the U.S. population in 1900 is closest to __. (A) 1 to 4 (B) 2 to 7 (C) 2 to 1 (D) 3 to 1 (E) 11 to 3

Answer: (E, 11 to 3) According to the graph, the U.S. population in 2000 was a little bit more than 275 million, and the U.S. population in 1900 was a little over 75 million. Since the question asks what the ratio is "closest to," these numbers are good enough to approximate. 275 to 75 can be reduced by 5 to get 55 to 15, which can be reduced by 5 again to get 11 to 3. Alternatively, you could reduce 275 to 75 by 25 to get this same ratio.

Question 2-2 The U.S. population in 1950 was approximately __ of the U.S. population in 1850. (A) 800% (B) 600% (C) 200% (D) 85% (E) 15%

Answer: (B, 600%) The question asks what percent the U.S. population in 1950 is of the U.S. population in 1850. To get this you need to calculate population 1950 ⁄ population 1850 x 100. Since the U.S. population in 1950 is higher, you want something that is greater than 100%. Eliminate 85% and 15%. Since the sentence says "approximate" and also since the remaining answer choices are not close to each other, you can estimate the values. According to the chart, the population in 1950 was about 150 million and the population in 1850 was about 25 million. Therefore, you need to calculate 150 ⁄ 25 x 100 = 6 x 100 = 600%.

Question 2-3 The U.S. population increased by approximately __ from 1900 to 1950. (A) 25% (B) 33% (C) 50% (D) 100% (E) 200%

Answer: (D, 100%) To get percent increase, you need to use the formula difference ⁄ original x 100. The population in 1900 was about 75 million, and the population in 1950 was about 150 million. The difference between the two figures is 75 million. Therefore, the percent increase is 75 ⁄ 75 x 100 = 100%.

Below you'll find a sample Analytical Writing Assessment (AWA) question. On the GMAT you'll have 30 minutes to write a critique of the argument.

Analysis of an Argument

The following appeared as part of a medical advertisement in a magazine.

A new medical test that allows the early detection of a particular disease will prevent the deaths of people all over the world who would otherwise die from the disease. The test has been extremely effective in allowing doctors to diagnose the disease six months to a year before it would have been spotted by conventional means.

Discuss how logically convincing you find this argument. In explaining your point of view, be sure to evaluate the line of reasoning and the use of evidence in the argument. For example, it may be necessary to consider what questionable assumptions underlie the thinking and what other explanations or counterexamples might weaken the arguments conclusion. You can also discuss what kind of evidence would strengthen or refute the argument, what changes in the argument would make it more logically persuasive, and what, if anything, would enable you to better evaluate its conclusion.

How will you score?

Take a GMAT practice test with us under the same conditions as the real thing. You'll get a personalized score report highlighting your strengths and areas of improvement.

START A FREE PRACTICE TEST

  • GMAT  

Featured Business Schools For You

Find MBA Programs Matched to Your Interests

Explore our featured business schools to find those that are looking for students like you.

Best Online MBA seal

Top Online MBA Programs

On a mission to increase your salary? Our Top 50 Online MBA ranking is based on academics, career outcomes, tech platforms, and more.

Best Career Prospects

Best Career Prospects

Find out which schools have the best track records for getting students jobs—and the highest starting salaries.

Top Entrepreneurship 2023 seal

Top Schools for Entrepreneurship

Ready to build your own business from the ground up? Check out these 50 graduate programs.

gmat example essay prompts

Free MCAT Practice Test

Thank you! Look for the MCAT Review Guide in your inbox.

I already know my score.

gmat example essay prompts

Enrollment Advisor

1-800-2REVIEW (800-273-8439) ext. 1

1-877-LEARN-30

Mon-Fri 9AM-10PM ET

Sat-Sun 9AM-8PM ET

Student Support

1-800-2REVIEW (800-273-8439) ext. 2

Mon-Fri 9AM-9PM ET

Sat-Sun 8:30AM-5PM ET

Partnerships

  • Teach or Tutor for Us

College Readiness

International

Advertising

Affiliate/Other

  • Enrollment Terms & Conditions
  • Accessibility
  • Cigna Medical Transparency in Coverage

Register Book

Local Offices: Mon-Fri 9AM-6PM

  • SAT Subject Tests

Academic Subjects

  • Social Studies

Find the Right College

  • College Rankings
  • College Advice
  • Applying to College
  • Financial Aid

School & District Partnerships

  • Professional Development
  • Advice Articles
  • Private Tutoring
  • Mobile Apps
  • Local Offices
  • International Offices
  • Work for Us
  • Affiliate Program
  • Partner with Us
  • Advertise with Us
  • International Partnerships
  • Our Guarantees
  • Accessibility – Canada

Privacy Policy | CA Privacy Notice | Do Not Sell or Share My Personal Information | Your Opt-Out Rights | Terms of Use | Site Map

©2024 TPR Education IP Holdings, LLC. All Rights Reserved. The Princeton Review is not affiliated with Princeton University

TPR Education, LLC (doing business as “The Princeton Review”) is controlled by Primavera Holdings Limited, a firm owned by Chinese nationals with a principal place of business in Hong Kong, China.

FB

Analytical Writing Assessment

GMAT Analytical Writing Assessment

You can download for free the list of current Argument essay prompts used on the GMAT. This PDF file is provided by the GMAC. Although it is lengthy, we recommend that you read through the entire list of essay topics in advanced of taking the GMAT. You should also pick at random at least 20 topics and spend five minutes brainstorming and outlining an essay. Furthermore, we suggest that you pick at random at least five more topics and spend 30 minutes writing out a full essay.

GMAT Analytical Writing Approach and Scoring

Your essay is judged on how well-structured and well-supported it is, and to a lesser extent on your usage of standard written English. The essay should be written in a straightforward, somewhat formulaic manner. A creative, nuanced writing style will not be rewarded. You must type your essay into the bare-bones word processor provided. Copy, cut, paste, redo, and undo are the only functions available. We suggest that you spend a few minutes brainstorming and outlining your essay before you begin writing. Also be sure to leave a few minutes at the end to proof-read your essay.

The AWA section does not factor whatsoever into your main 200-800 GMAT score. You get a separate AWA score on a scale from 0 to 6 , in half-point increments. Your essay is graded separately by two readers: a college faculty member and a computer scoring program. If the two scores are within one point of each other, as is typically the case, the two scores are averaged to derive your AWA score. If the two scores are more than one point apart, however, another human reader will evaluate your essay. The AWA section is not particularly difficult . In fact, 13% of test takers score a perfect 6.0 and 82% score 4.0 or above. Because of this, most business schools primarily use the AWA score to ensure that foreign students can write well and to verify that the essays included on your business school applications are consistent with your AWA writing abilities.

Sample GMAT Analytical Writing Essay Topic

Let’s try a sample Argument essay topic. Brainstorm flaws on your own and perhaps even write out a full essay before viewing the explanation and sample essay.

The following appeared as part of an article in the business section of a local newspaper:

“Ronnie’s Auto Body Shop commenced business four months ago at the location formerly occupied by the Mystique Beauty Parlor. Ronnie’s Auto must be doing well at this location, because it intends to open a big body shop in an adjacent town. Mystique, on the other hand, has seen a lower volume of business in its first year at its new location compared to the prior year at its former location. Mystique definitely erred in shifting to its new location; its former location is a better site.”

Discuss how well reasoned you find this argument. In your discussion be sure to analyze the line of reasoning and the use of evidence in the argument. For example, you may need to consider what questionable assumptions underlie the thinking and what alternative explanations or counterexamples might weaken the conclusion. You can also discuss what sort of evidence would strengthen or refute the argument, what changes in the argument would make it more logically sound, and what, if anything, would help you better evaluate its conclusion.

Explanation to Problem

As with all Argument essay prompts, this prompt contains several flaws. Let’s briefly outline the problems with this argument before presenting a full essay that encompasses these points. The sample essay that follows would receive a top score of 6.

  • Illogical comparison between Ronnie’s Auto Body Shop and Mystique Beauty Parlor. These are very different types of businesses, and a good location for one may be a poor location for the other.
  • Assumes that Ronnie’s Auto Body Shop has been successful, but offers insufficient evidence for this. Ronnie’s Auto may be opening a big body shop in an adjacent town because the current location is poor.
  • Inappropriately concludes that Mystique erred in shifting locations. Once again, offers insufficient evidence for this. Lower volume of business is vaguely defined, may not be a long-term concern, and perhaps is not even relevant.

Sample Essay

The author offers a flawed argument in claiming that Mystique erred in shifting locations and that its former location is a better site. The author offers insufficient evidence for these claims while providing an illogical comparison. In particular, the author assumes that Ronnie’s Auto Body Shop has been successful in Mystique’s former location. Second, the author overlooks the differences in the two business types. Third, the author inappropriately equates a lower volume of business to a poor decision by Mystique regarding location. Let’s elaborate on each of these issues in turn.

One problem in the argument is that the author assumes that Ronnie’s Auto is doing well at its current location. The only evidence that the author offers in support of this claim is that Ronnie’s Auto intends to open a big body shop in an adjacent town. But there are many possible reasons as to why Ronnie’s Auto plans to open a large shop in another town. Most notably, perhaps the existing location is performing poorly and Ronnie’s Auto plans to close this location after opening up elsewhere.

A second problem in the argument is the apples-to-oranges comparison between an auto body shop and a beauty parlor. The customer base for the two businesses is probably very different. Even if Ronnie’s Auto is doing well at its current location, a totally different type of business with a totally different customer base would not necessarily do well at the same location. Perhaps Ronnie’s Auto is located in a commercial area with similar types of businesses (e.g. other auto repair and parts businesses). Mystique Beauty Parlor is unlikely to thrive in such a location, since its customers would probably not want to receive beauty treatments in such an area.

Finally, a third problem is that the author concludes based solely on “lower volume of business” that Mystique erred in shifting locations. But how much lower is Mystique’s volume of business in the new location? Perhaps the drop in business volume is very small. In fact, we would expect some drop when moving to a new location, as it takes time to reestablish a customer base. The new location may hold much greater long-term promise, and future business volume may be much higher. We also do not know what volume of business really means. By volume, does the author mean customers or revenue? Either way, a lower volume of business does not necessarily mean lower profits. Perhaps Mystique’s lease and other costs are lower at the new location, resulting in higher profits. If so, we cannot claim that Mystique make a mistake in moving. But even if the move did result in lower profits, the move was not a mistake unless Mystique’s owners have this as a primary concern. Perhaps profit is secondary to Mystique’s owners, and they moved the beauty parlor to a location that affords them other benefits such as a better quality of life.

In conclusion, the author’s claims that Mystique erred in shifting locations and that its former location is better are unsupported. As discussed above, there are far too many unknown variables for the author to make such a definite conclusion. Had the author considered the points discussed above, the argument could have been presented more thoughtfully.

Prev

whatsapp

  • GMAT Classroom Training
  • GMAT Quant Private Tutoring
  • GMAT Verbal Private Tutoring
  • GMAT Blend – Self Paced with Teacher Access
  • GMAT Sample Paper
  • GMAT Practice Test
  • GMAT Vocabulary
  • GMAT Exam Dates 2024
  • GMAT Exam Pattern 2024
  • GMAT Syllabus 2024
  • GMAT Exam Eligibility Criteria 2024
  • GMAT v/s CAT

About the GMAT Exam

  • GMAT Course
  • GMAT Verbal Reasoning
  • GMAT Quantitative Reasoning
  • GMAT Integrated Reasoning
  • GMAT Analytical Writing
  • Apply for GMAT
  • GMAT Quant 1 on 1
  • GMAT Verbal 1 on 1

GMAT Practice Material

  • GMAT Test Series
  • GMAT Vocab Builder
  • GMAT Exam Dates 2021
  • GMAT Exam Pattern 2021
  • GMAT Syllabus 2021
  • GMAT Eligibility 2021
  • How to Prepare for the GMAT

GMAT Registration

Gmat analytical writing assessment.

The Analytical Writing Assessment (AWA) section of the GMAT contains one essay prompt and is 30 minutes long. The AWA tests your ability to analyze a logical argument and then write a coherent and convincing essay, evaluating the argument.

The topic on the argument essay can be extracted from a wide range of topics related to business. However, prior knowledge about the topic would not be required. The AWA only tests your ability to analyze the argument and write a cogent essay.

The AWA score would be published within 20 calendar days from the day of your exam and would be a part of your Official GMAT Score Report. If you feel that the score obtained on the AWA does not reflect your performance, you can choose to request rescoring your essay. Rescoring the essay is possible only once and requires additional payment. For rescoring, you need to make a request within 6 months from the date of your exam. AWA rescoring requests can result in a score increase or decrease.

Topics Tested on the GMAT Analytical Writing Assessment Section

The AWA section tests a candidate’s ability to analyze a given argument, identify the inherent flaws in it, and provide logical and reasonable solutions in the form of an essay. The clarity in reasoning and coherence of thought are major assessment criteria for the AWA. The essay also needs to conform to elements of standard written English.

More About GMAT Related Services

  • Admissions Consulting |
  • MBA Brochure

GMAT Analytical Writing Assessment Scoring Pattern

The Analytical Writing Assessment section is scored on a range of 0 to 6, in increments of 0.5 points.

The essay is evaluated by a human and also by a machine algorithm. If there is a difference between the way the human and the machine have scored your essay, a second human evaluator is brought in, and your essay score may be adjusted. You will receive only one essay score, which is an average of the evaluations.

How Would You Do If You Took The GMAT Today? Click Here To Take Free Practice Test

Analytical writing assessment comparison, our top gmat achievers.

  • Sooraj Kapur
  • Bengaluru | WHT181906E0023
  • Jyotika Khatri
  • Gurgaon | GD4181901E0030
  • Kunal Bhagi
  • Bengaluru | INR171807E00001
  • Bengaluru | YLK171810E0018

Testimonials

Harshit Singh Harshit Singh Through this testimonial, I would like to provide an insight into my GMAT prep and how important a role the Princeton Review played in enabling me to score a 740 on my GMAT and that too in my first attempt. I got to know about the Manya Princeton Review through a friend. Although initially, I had my reservations as Princeton Review is not as heavily advertised as some of the other MBA training institutes, especially in India, all my doubts came to rest once I visited the website and got to know about the whole structure of the program. I started my online classes with the Manya Princeton Review around mid-Jan 2022. I was almost awestruck by the online portal where you could have access to unlimited study material in the form of e-books, topic-wise practice drills, practice tests, etc. But the most important feature that helped me personally was the unlimited access to the recorded classes which enabled me to revise anything which I missed or was not able to fully understand during the live classes. These class recordings also served as a great revision tool, especially for the verbal section, and helped me to get my basics firmly in place. The teaching techniques used by the Princeton faculty were also really incredible and easy to comprehend . In particular, the strategies taught by Princeton’s faculty enabled me to start thinking out of the box and somewhat predict as to what the test takers are actually trying to test through particular questions in both Verbal & Quant sections. Additionally, I would also like to give a special mention to my Booster trainers (Raji Ma’am & Seenu sir) . After completing my 1.5 months of classes and fulfilling some other criteria I was able to qualify for the Booster sessions or classes which the Princeton Review provides absolutely free of cost. The personalised booster sessions proved absolutely critical for my GMAT prep enabling me to identify and improve on my weaknesses. To be honest, it was these booster sessions that fine-tuned my performance and raised my level from a 650-680 (scores which I was getting in my mocks) to a 730+ score which was my goal when I started my GMAT prep. Finally, I would like to thank the entire Manya Princeton Review team(teaching faculty, IT dept., Admissions team, etc.) for their highly valued guidance and co-operation. I would forever be indebted for the role Princeton Review played in my GMAT prep and my entire career as a whole.
Keshav Maheshwari Keshav Maheshwari Manya - Princeton Review provided me with the knowledge necessary to achieve my target score & I am grateful that I chose it. The teachers were extremely approachable throughout my preparation period. They frequently inquired about my progress & which helped me stay motivated. Besides that, they were always available on call or text, ready to clear any doubts I had.
Sachet Garg Sachet Garg My experience with GMAT preparation at Manya - The Princeton Review was great. The teachers are very experienced, the online portal contains 10 practice tests & a lot of topic-wise drills for practice, which makes it easy to customize our preparation according to our preferences. The classes taught us a lot of techniques & the booster sessions were especially helpful as they helped with difficult questions. Overall, it really helped me improve my score to 740 & I would recommend preparation from Manya - The Princeton Review to everyone.
Shivreet Majitha Shivreet Majitha Manya - The Princeton Review has helped me reach my goal. It was in Dec 2020 that I decided to join Manya – The Princeton Review & realised that there were so many tips & tricks that I had no idea about & I got 740. The online 1:1 classes & the additional booster sessions helped me strengthen my basics. Additionally, the mock-up review helped me understand my weak areas and work on them. I would like to thank the teachers who ensured that I understood the concepts & were deeply involved in my progress.
Rudra Mishra Rudra Mishra My journey started with knowing little to nothing about the GMAT but ended with me scoring a 760 in the first attempt. During this time, the staff at Manya – The Princeton Review helped me understand what the GMAT actually was, how it was different from other standardized tests & the approach one needed to take to score well. Like everyone, I too wanted to have an idea of where I stood before I gave the actual test. I used the practice tests from the GMAT website as well as those offered by Manya to get that idea. The study resources, performance analysis and timely motivation and advice provided by Manya – The Princeton Review helped me achieve a good score.

gmat example essay prompts

gmat example essay prompts

gmat example essay prompts

What is the GMAT analytical test?

In the Analytical Writing Assessment section or the essay section, you have to analyze an argument, identify the flaws in the argument and write an essay about it. Prior specific knowledge is not required.

Is analytical writing important in GMAT?

Analytical writing assessment (AWA)is important on the GMAT. Even though the AWA does not contribute to the composite GMAT score ( 200-800), it is used by different universities and departments during the application selection process. The writing skills tested in the AWA would prove beneficial even during your actual course as many programs require students to submit essays as a part of their coursework.

What should I study for GMAT Analytical?

In the Analytical Writing Assessment section or the essay, you have to analyze an argument, identify the flaws in the argument and write an essay about it. Prior specific knowledge is not required. You should practice evaluating arguments, understanding and working using relevant templates for these essays and writing the essays, all within 30 minutes (time available for this section on the GMAT).

What is the syllabus for the GMAT Analytical section?

How do i prepare for gmat analytical writing, gmat trending articles.

GMAT Preparation Books: Mastering the Exam with Essential Guides

GMAT Preparation Books: Mastering the Exam with Essential Guides

The GMAT preparation books - stepping stones to success in GMAT A B-school aspirant has to ... Read More >

Best 10 Business Schools and MBA Degrees in the UK 2023 Blog Banner

Best 10 Business Schools and MBA degrees in the UK 2023

For international students pursuing MBA at the top business ... Read More >

GMAT Score Chart 2023

GMAT Score Chart 2023 – A Comprehensive Guide for GMAT Aspirants

The GMAT (Graduate Management Admission Test) is a standardized ... Read More >

MBA at ISB Hyderabad

Admission Process for MBA at ISB Hyderabad

The Indian School of Business (ISB) holds a strong ... Read More >

Google Reviews

Shobana ramesh ().

gmat example essay prompts

If you're looking for the best Institute for GRE or GMAT , this is the place you have to reach out. Friendly Faculty, Easily Understandable Methods. Boosted my Confidence Level to Clear my GMAT. This review is my way of Thanking them.

If you’re looking for the best Institute for GRE or GMAT , this is the place you have to reach out. Friendly Faculty, Easily Understandable Methods. Boosted my Confidence Level to Clear my GMAT. This review is my way of Thanking them.

PRITISH KAR (Nagpur)

It is good. Faculty are nice and the tips and tricks required specifically for GMAT are taught.

Sapna Singh (hyderabad)

It's best in all prospectives although I tried the GMAT demo and it was super

It’s best in all prospectives although I tried the GMAT demo and it was super

GMAT Coaching Centers

  • Bhubaneswar
  • ">Mangalore
  • Vishakhapatnam

Upcoming Webinars/Events

gmat example essay prompts

  • 5:30 - 6:30 PM

Pursue Master's Degree in the UK Canada Australia

Join us as we dive into the dynamic shifts in the current education system across countries like the UK, Canada, and Australia.

Address: PAN INDIA

Call

Just One Step Away!

Our Experts require more information to assist you in a better way.

Testimonial

Privacy Overview

Necessary cookies are absolutely essential for the website to function properly. This category only includes cookies that ensures basic functionalities and security features of the website. These cookies do not store any personal information.

Any cookies that may not be particularly necessary for the website to function and is used specifically to collect user personal data via analytics, ads, other embedded contents are termed as non-necessary cookies. It is mandatory to procure user consent prior to running these cookies on your website.

GMAT essay topics: Expert tips for a high-scoring performance

The GMAT essay, also known as the Analytical Writing Assessment (AWA), is a section of the GMAT exam that can often be intimidating for test-takers. However, with the right preparation and understanding of the essay topics, test-takers can excel and achieve a high score.

The AWA section is designed to evaluate the test-taker's ability to analyse arguments, express ideas clearly and concisely, and support their viewpoints with evidence. This section consists of two types of essay prompts, an Analysis of an Issue and an Analysis of an Argument, each of which requires specific skills and strategies.

In this article, we will provide expert examples of both types of GMAT essay topics and tips on how to approach them to help you achieve a high-scoring performance in the AWA section.

Table of Contents

1. analysis of an argument, 2. analysis of an issue, gmat essay topics for analysis of an argument, gmat essay topics for analysis of an issue, gmat essay practice questions, 1. gmat essay topics for analysis of an argument, 2. gmat essay topics for analysis of an issue, 1. understand the prompt and task, 2. plan your essay, 3. analyse the argument or issue, 4. use specific examples, 5. write clearly and concisely, 6. follow the standard essay format, 7. practice, practice, practice, what is the gmat essay, how is the gmat essay scored, how do i prepare for the gmat essay.

The GMAT essay, also known as the Analytical Writing Assessment (AWA), is a section of the GMAT exam that requires you to write two essays within a total of 60 minutes.

There are two types of essay prompts in the GMAT AWA section:

  • Analysis of an Argument
  • Analysis of an Issue

In this type of essay prompt, you will be given an argument that you need to analyse and critique. You will be required to identify the assumptions that the argument makes and evaluate the evidence that it provides. You will need to explain why the argument is flawed or weak and suggest ways to improve it.

In this type of essay prompt, you will be given a statement or question related to a social or political issue, and you will be asked to take a stance on the issue and support your position with evidence and examples. You will need to provide a clear argument and present evidence that supports your viewpoint.

Also read: 5 IELTS Sample Essays

GMAT Analytical Writing Section

The GMAT Analytical Writing Assessment section assesses your ability to analyse arguments and communicate your thoughts effectively in written form. In this section, you are presented with an argument or an issue prompt and are required to critically evaluate and provide a well-structured response within a given time frame.

Although the AWA section is not as heavily weighted as the Quantitative and Verbal sections, it still plays a role in showcasing your analytical abilities and written communication skills to business schools. A well-written essay in the AWA section can leave a positive impression on the admissions committee and contribute to your overall application.

  • The construction of a new highway through the rural town of Pineville is essential to improve connectivity and drive economic growth. Discuss the validity of this argument and evaluate its potential impact on the community.
  • The introduction of a mandatory recycling program in our city is necessary to combat the growing environmental concerns and promote sustainable practices. Analyse the reasoning behind this argument and assess its potential effectiveness.
  • Expanding the public library's operating hours in the urban neighbourhood of Riverton will provide greater access to educational resources and foster a culture of lifelong learning. Evaluate the logic behind this proposal and its potential benefits for the community.
  • Establishing stricter regulations on the use of single-use plastics is crucial to mitigate the environmental damage caused by plastic pollution. Assess the reasoning behind this argument and discuss its potential impact on businesses and consumers.
  • The implementation of a tax incentive program to encourage the adoption of solar energy systems in residential areas will promote renewable energy usage and reduce carbon emissions. Evaluate the logic behind this proposal and its potential long-term effects.
  • Constructing a new sports stadium in the city centre will not only revitalise the local economy but also enhance the city's reputation as a sports destination. Analyse the soundness of this argument and its potential implications for the community.
  • Implementing stricter gun control measures is necessary to address the rising concerns of public safety and reduce gun-related violence. Evaluate the reasoning behind this argument and discuss the potential challenges associated with its implementation.
  • The introduction of a comprehensive sex education curriculum in schools is vital to promote safe and responsible behaviour among adolescents. Assess the validity of this argument and discuss the potential benefits and drawbacks of such an initiative.
  • Investing in the development of affordable housing units in urban areas is essential to address the housing crisis and ensure social equity. Analyse the rationale behind this proposal and evaluate its potential impact on both residents and the local economy.
  • The implementation of stricter regulations on the advertising and sale of unhealthy food products will contribute to improving public health and combating the obesity epidemic. Discuss the validity of this argument and its potential implications for businesses and consumers.
  • Increasing funding for arts and culture programs in schools can have a positive impact on students' creativity, critical thinking skills, and overall academic performance. Evaluate the logic behind this proposal and discuss its potential benefits for the education system.
  • Enforcing mandatory financial literacy courses in high schools will equip students with essential skills to make informed decisions and achieve long-term financial stability. Assess the reasoning behind this argument and discuss the potential outcomes of such an educational initiative.
  • The adoption of telecommuting policies in organisations can lead to improved work-life balance, increased employee satisfaction, and enhanced productivity. Analyse the reasoning behind this proposal and discuss its potential advantages and challenges for both employers and employees.
  • Investing in the development of public parks and green spaces in urban areas can have numerous benefits, including improved mental and physical well-being, enhanced community cohesion, and increased property values. Evaluate the soundness of this argument and its potential impact on urban environments.
  • Introducing a universal basic income program can help alleviate poverty, reduce income inequality, and provide individuals with a safety net in an increasingly automated world. Analyse the reasoning behind this proposal and discuss its potential implications for society and the economy.
  • Discuss the impact of social media on interpersonal communication and relationships.
  • To what extent should governments prioritise funding for space exploration?
  • Explain your views on the role of artificial intelligence in shaping the future of work.
  • Discuss the ethical considerations of genetic engineering and its potential impact on society.
  • Explain your perspective on the effectiveness of affirmative action policies in promoting equality.
  • To what extent should the government regulate the sharing economy platforms such as Uber and Airbnb?
  • Discuss the benefits and drawbacks of the globalisation of markets and its impact on local economies.
  • Explain your views on the role of government in promoting sustainable development.
  • Discuss the ethical implications of data privacy and the use of personal information by technology companies.
  • Explain your perspective on the importance of arts and humanities education in today's society.
  • Discuss the impact of climate change on global food security and the role of international cooperation.
  • Explain your views on the ethical considerations surrounding animal testing in scientific research.
  • Discuss the advantages and disadvantages of remote work for employees and organisations.
  • Explain your perspective on the role of the government in regulating the use of social media platforms.
  • Discuss the implications of income inequality and the role of government in addressing this issue.

GMAT practice questions provide an opportunity to develop and refine your critical thinking, analytical, and writing skills. By engaging with a variety of prompts, you can familiarise yourself with different argument structures, evaluate evidence, and effectively express your ideas within the given time constraint.

We have listed ten GMAT sample questions that cover a range of topics and argument types to challenge you to analyse arguments, identify flaws, consider counterarguments, and develop well-structured responses.

GMAT sample question #1

The following appeared in a company memo:

Our new advertising campaign was a great success. The number of customers visiting our website increased by 50% in the first month after the campaign's launch. Therefore, the campaign should be credited with attracting new customers and boosting sales.' Discuss how well-reasoned you find this argument. In your discussion, analyse the argument's assumptions and evidence.

GMAT sample question #2

The following appeared in a research paper:

Implementing a four-day workweek will significantly enhance employee productivity. A survey conducted among companies that adopted a shorter workweek showed a 20% increase in employee output and a 15% decrease in absenteeism rates. Therefore, all organisations should consider adopting a four-day workweek to boost efficiency.' Discuss how well-reasoned you find this argument. In your discussion, analyse the argument's assumptions and evidence.

GMAT sample question #3

The following appeared in a political campaign speech:

Increasing the minimum wage will improve the economy and reduce income inequality. Studies have shown that when the minimum wage is raised, low-income workers experience a rise in their earnings, leading to increased consumer spending. As a result, businesses thrive, unemployment rates decrease, and the wealth gap narrows. Therefore, it is imperative to raise the minimum wage to stimulate economic growth.' Discuss how well-reasoned you find this argument. In your discussion, analyse the argument's assumptions and evidence.

GMAT sample question #4

The following appeared in a blog post:

Investing in early childhood education is crucial for fostering future academic success. Research has consistently shown that children who receive quality early education perform better academically and are more likely to graduate from high school and pursue higher education. Therefore, policymakers should prioritise funding and expanding early childhood education programs.' Discuss how well-reasoned you find this argument. In your discussion, analyse the argument's assumptions and evidence.

GMAT sample question #5

The following appeared in a marketing presentation:

Social media marketing is the most effective strategy for reaching and engaging with the target audience. A survey conducted among consumers revealed that 75% of respondents made a purchase based on social media ads or recommendations from influencers. Therefore, companies should allocate a significant portion of their marketing budget to social media campaigns to maximise their reach and sales.' Discuss how well-reasoned you find this argument. In your discussion, analyse the argument's assumptions and evidence.

GMAT sample question #6

Prompt: Evaluate the argument that increasing taxes on sugary beverages will effectively reduce the prevalence of obesity and improve public health. Discuss the key assumptions and evidence used to support this claim, and identify any potential counterarguments or limitations that may challenge the effectiveness of this approach.

In your analysis, you should consider the relationship between sugary beverage consumption and obesity rates, as well as the potential impact of higher taxes on consumer behaviour. Additionally, assess the strength of the evidence presented to support the argument and examine any potential flaws or biases in the data. Consider alternative approaches to tackling obesity, such as education campaigns or regulations on advertising, and discuss their potential advantages or disadvantages. Ultimately, evaluate the argument's overall persuasiveness and suggest any additional evidence or considerations that could strengthen or weaken the case for increased taxes on sugary beverages.  

GMAT sample question #7

Prompt: Analyse the argument that social media platforms should be held legally responsible for moderating user-generated content. Evaluate the reasons and evidence presented to support this viewpoint, and discuss the potential consequences of imposing legal obligations on these platforms. Consider issues such as freedom of speech, platform liability, and the challenges of content moderation at scale. Additionally, explore alternative approaches to addressing harmful or illegal content online and discuss their potential advantages or drawbacks. Finally, assess the feasibility and potential effectiveness of different regulatory or self-regulatory measures in ensuring a safe and responsible online environment.  

GMAT sample question #8

Prompt: Evaluate the argument that increasing access to higher education will lead to greater socioeconomic mobility and reduce income inequality. Analyse the evidence and reasoning used to support this claim, and consider potential counterarguments or limitations to this approach. Assess the impact of factors such as rising tuition costs, disparities in educational quality, and the relevance of certain degrees in the job market. Explore alternative strategies for promoting social mobility, such as vocational training or income redistribution, and discuss their potential advantages or disadvantages. Ultimately, critically examine the argument's validity and provide recommendations or additional considerations that may strengthen or weaken the case for increasing access to higher education.

GMAT sample question #9

Prompt: Analyse the argument that the use of genetically modified organisms (GMOs) in agriculture is essential for ensuring global food security. Evaluate the evidence and reasoning presented to support this claim, and discuss any potential counterarguments or concerns related to the environmental, health, or socioeconomic impacts of GMOs. Assess the effectiveness of current regulations and labelling practices in addressing these concerns, and explore alternative approaches to achieving food security, such as organic farming or agroecology. Finally, critically evaluate the overall persuasiveness of the argument and provide recommendations or additional considerations that may enhance or challenge the case for GMO usage.

GMAT sample question #10

Prompt: Evaluate the argument that implementing a universal basic income (UBI) would be an effective solution to address poverty and income inequality. Analyse the evidence and reasoning used to support this viewpoint, and consider potential counterarguments or limitations of a UBI system. Assess the economic implications, including funding mechanisms and potential effects on workforce participation and productivity. Explore alternative policies or approaches to poverty alleviation, such as targeted welfare programs or job creation initiatives, and discuss their potential advantages or disadvantages. Ultimately, critically examine the argument's validity and provide recommendations or additional considerations that may strengthen or weaken the case for implementing a universal basic income.

GMAT essay topics

Some common topics that may be covered in the GMAT essays include:

  • The research indicates that the success of a shopping mall is directly related to its location. Therefore, we should build a new shopping mall in the small town of Oakville, which has a growing population and no competition from other shopping malls. Discuss how well-reasoned you find this argument.
  • To remain competitive, companies need to embrace new technologies, including artificial intelligence and automation. Employees who resist these changes are hindering progress and putting their jobs at risk.' Evaluate the argument presented in the editorial.
  • Our company should offer tuition reimbursement to all employees who want to pursue a degree or certification. This will increase employee loyalty, productivity, and skill level, which will benefit the company in the long run.' Discuss the validity of this recommendation.
  • Our new cleaning products are environmentally friendly and safe for use around children and pets. You can trust our products to not only clean effectively, but also protect your family and the planet. Evaluate the claims made in the advertisement."
  • The city should ban all smoking in public parks. Secondhand smoke is a health hazard to all park visitors, and it is unfair to subject non-smokers to this danger. Evaluate the argument presented in the letter.

Also read: TOEFL sample essays

  • It is better to be practical than to be creative when it comes to problem-solving.
  • Governments should prioritise funding for space exploration over other scientific endeavours.
  • The widespread use of social media has had a negative impact on society.
  • The pursuit of happiness should be considered a fundamental human right.
  • The use of technology has had a more positive than negative impact on society.
  • Businesses have a responsibility to prioritise environmental sustainability over profit.
  • The government should provide free healthcare for all citizens.
  • The increasing popularity of e-books will eventually lead to the end of traditional bookstores.
  • Individuals have a responsibility to prioritise their own health and well-being over their career ambitions.
  • The benefits of globalisation outweigh the drawbacks.

Also Read: GMAT Self-Study Plan

Tips for a high-scoring performance

Here are some expert tips to help you achieve a high-scoring performance on the GMAT essay.

The first step to writing a successful GMAT essay is to understand the prompt and the task. Take the time to read the prompt carefully and make sure you understand what is being asked of you. Pay close attention to the specific instructions, such as the length requirement, the number of examples you need to provide, and the time limit.

Before you start writing, take a few minutes to plan your essay. This will help you organise your thoughts and ensure that your essay is well-structured and easy to follow. Create an outline that includes your main points and supporting evidence, and make sure that your essay has a clear introduction, body, and conclusion.

For the Analysis of an Argument task, carefully analyse the argument presented in the prompt. Identify the assumptions and evidence used to support the argument, and consider whether these are valid and sufficient. For the Analysis of an Issue task, consider both sides of the issue and identify the strengths and weaknesses of each argument.

To support your arguments, use specific examples from your own experiences or the world around you. These examples should be relevant and support the points you are making in your essay. Be sure to explain how each example supports your argument.

The GMAT essay is not a test of your writing ability per se, but rather a test of your ability to analyze and evaluate arguments and express your ideas in a clear and concise manner. Use simple and straightforward language, avoid overly complex sentences or jargon, and be sure to proofread your essay for grammatical errors and spelling mistakes.

The GMAT essay should follow a standard essay format, with a clear introduction, body, and conclusion. Your introduction should provide an overview of your argument, while your body should provide supporting evidence and examples. Your conclusion should summarise your main points and restate your thesis statement concisely.

Like any skill, the ability to write a successful GMAT essay takes practice. Take advantage of the practice exams and sample essays provided by GMAC, the organisation that administers the GMAT exam. Consider working with a tutor or taking a prep course to help you improve your essay writing skills.

Articles related to GMAT Exam

  • GMAT Preparation Books
  • GMAT Grammer
  • GMAT Integrated Reasoning Tips
  • Preparing for MBA Without GMAT  
  • GMAT Vocabulary
  • GMAT Sentence Correction
  • GMAT Critical Reasoning
  • Online GMAT Preparation Tips
  • GMAT Reading Comprehension
  • GMAT Math Formulas
  • GMAT Customized Study Plan
  • GMAT Geometry
  • GMAT Test Resources

The GMAT essay is an important part of the GMAT exam and requires careful preparation and planning. By following these expert tips, you can improve your chances of achieving a high score on the GMAT essay and increase your chances of being accepted into your dream MBA program.

Frequently asked questions

The GMAT essay, also known as the Analytical Writing Assessment (AWA), is a section of the GMAT exam that assesses a candidate's ability to think critically and communicate ideas in written form.

The GMAT essay is scored on a scale of 0-6 in half-point increments. Two independent readers will score the essay, and if their scores differ by more than one point, a third reader will be brought in to resolve the discrepancy.

To prepare for the GMAT essay, candidates can practise writing essays using free GMAT essay prompts available online, develop a plan before writing, use specific examples to support their arguments and practise good grammar and syntax.

author avatar

Study Abroad Expert

Disclaimer: The views and opinions shared in this site solely belong to the individual authors and do not necessarily represent t ...Read More

The ultimate guide to mastering the GMAT exam

The ultimate guide to mastering the GMAT exam

GMAT vs GRE - Which is easier?

GMAT vs GRE - Which is easier?

6 best GMAT prep books that students swear by

6 best GMAT prep books that students swear by

GMAT vs CAT: Key differences and similarities

GMAT vs CAT: Key differences and similarities

Mastering the GMAT integrated reasoning section: Expert tips

Mastering the GMAT integrated reasoning section: Expert tips

How to prepare for an MBA program without the GMAT

How to prepare for an MBA program without the GMAT

404 Not found

GMAT Prep Online Guides and Tips

The 6 gmat essay tips that will help you ace the awa.

gmat example essay prompts

If you’re preparing for the GMAT, you’ve probably spent countless hours reviewing math concepts and mastering grammar skills. You’ve likely also spent time studying for the newer integrated reasoning section, too. But have you thought about the analytical writing assessment part of the GMAT?

If your answer is no, don’t worry! You’re not alone. Many test-takers go into test day without spending a lot of time preparing for the essay section of the GMAT, especially since it’s unclear how much (or even if) the GMAT essay even matters for getting into business school.

In this article, I’ll shed some light on the oft-forgotten GMAT AWA section. First, I’ll give you an overview of what’s actually on the AWA section. Next, I’ll discuss whether or not that score really matters for your admission to business school. Finally, I’ll tell share  the top GMAT essay tips that are guaranteed to boost your GMAT essay score.

GMAT Analytical Writing Assessment Overview

The GMAT Analytical Writing Assessment is designed to measure your ability to think critically about a topic and then communicate your ideas about that topic. During the AWA section, you’ll be asked to analyze and critique an argument and judged on your ability to do so clearly, thoroughly, and thoughtfully.

The GMAT AWA section consists of one writing task: a 30-minute essay. You’ll complete the AWA portion of the GMAT first, before every other test section.

Not sure how or what to study? Confused by how to improve your score in the shortest time possible? We've created the only Online GMAT Prep Program that identifies your strengths and weaknesses, customizes a study plan, coaches you through lessons and quizzes, and adapts your study plan as you improve.

We believe PrepScholar GMAT is the best GMAT prep program available , especially if you find it hard to organize your study schedule and don't want to spend a ton of money on the other companies' one-size-fits-all study plans.

     Improve Your GMAT Score by 60 Points, Guaranteed     

For your GMAT essay, you’ll be asked to think critically about an argument that’s presented to you. You’re not supposed to give your opinion on the subject itself.

GMAT AWA scores range from 0 to 6, in half-point intervals. Every GMAT AWA response receives two independent scores. According to MBA.com, one of your scores may be performed by an essay-scoring engine. At least one of your GMAT AWA scores will be determined by a GMAT essay reader.

Your AWA score doesn’t affect your GMAT total score and is generally considered the least important of your GMAT scores.

control-427512__340

The 6 Best GMAT Essay Tips

If you’re looking to achieve a GMAT essay score that’ll help you get into business school, these six GMAT Analytical Writing tips will help you achieve success.

#1: Follow the Directions

One of the most important GMAT essay tips is to understand the directions of the AWA section.

The AWA section specifically asks you to critique an argument on its strengths and weaknesses. AWA graders aren’t looking for a well-written, thoughtful opinion piece about the topic discussed in the prompt. They’re looking for you to analyze whether or not the argument itself was sound, and to back up that analysis with evidence from the text, and they’ll judge you on how well you accomplished that specific task. If you don’t follow the directions, you won’t achieve a high score.

#2: Develop a Clear Structure

Another one of the important GMAT writing tips is to take the time to set up your essay in a clear way.

You don’t need to write the most interesting or lengthy essay in the world to score well on the AWA section, but you do need to give your essay an easy-to-follow structure. Usually, that consists of an introduction, three to four well-developed body paragraphs, and a conclusion.

Your introduction should restate the main argument of the prompt, then highlight the flaws in the argument that you’ll discuss in the body of the essay.

Each of the body paragraphs should focus on a specific flaw in the argument. First, you should highlight the flaw itself. Next, you’ll need to explain why that particular flaw is a flaw. Finally, you should highlight how the argument could’ve been made more clearly or more successfully.

In the conclusion, you’ll want to restate each of the reasons why the argument was flawed and summarize how those flaws affected the validity of the argument.

Following this clear, simple structure for your GMAT essay will help you achieve your goal score.

#3: Know the Common AWA Flaws

Your task for the GMAT AWA is to critique an argument given to you in a prompt. That means that you can assume the argument given is a weak one, since your job is basically to analyze its weaknesses.

GMAT AWA prompts typically have arguments that are weak in predictable ways. Be on the lookout for these common “flaws” that you’ll encounter in AWA prompts:

Causality: GMAT AWA prompts often contain errors in causality, which means that they attribute the wrong effect to the wrong cause . If you see an argument that uses causality, make sure you check to make sure that causality is correctly attributed and that there’s a provable causal relationship.

Vagueness: GMAT AWA prompts often contain vague terms or statistics that are used incorrectly to draw conclusions. For instance, a prompt might suggest that, out of a sample of 500 consumers, more are buying name-brand paper towels than generic paper towels. The use of the word “more,” in this case, isn’t specific enough because it doesn’t tell you exactly how many more people are buying name-brand paper towels. You can’t draw a definitive conclusion off of vague data.

Overconfidence: GMAT AWA prompts often contain overconfident language. You should be looking for the language in arguments to be thoughtful and well-balanced. Keep an eye out for words like “undoubtedly,” “definitely,” and “of course,” which indicate overconfidence.

home-office-336378__340

#4: Practice

One of the best GMAT essay tips is to practice, practice, practice before you actually complete the GMAT AWA section on test day. You can find real, retired GMAT AWA prompts on the GMAT website for free. You can also purchase the GMAT Write tool to receive scores on practice AWA prompts if you’re really concerned about your score.

Want to improve your GMAT score by 60 points?

We have the industry's leading GMAT prep program. Built by Harvard, MIT, Stanford, and Wharton alumni and GMAT 99th percentile scorers, the program learns your strengths and weaknesses and customizes a curriculum so you get the most effective prep possible.

Try PrepScholar GMAT for 5 Days Risk-Free.

Practicing will help you in a number of ways. First, practicing will help you master your timing. You’ll only have 30 minutes to craft a logical and well-reasoned essay on test day. The more you practice, the faster you’ll get at outlining and completing your essay.

As I mentioned in the previous GMAT writing tips, you’ll need to fully answer the correct prompt to achieve a good score on your GMAT essay. Practicing will help you get used to the structure of GMAT AWA prompts and help you get used to the types of questions you’ll see on test day.

Finally, practicing will help you get used the structure you need to employ to succeed on your GMAT essay. The more you practice, the more naturally you’ll be able to craft a complete introduction, body, and conclusion for each of your GMAT essays.

#5: Take Time to Outline

While outlining may seem like one of the more basic GMAT essay tips, taking five minutes at the beginning of the AWA section to sketch out a basic outline of your essay will really help you as you start to write.

Everyone outlines differently, but in general, I’d suggest having one to two bullet points for each paragraph that highlight the main ideas the paragraph will cover. Outlining will help you make sure you’ve covered all the main points you need to fully answer the question.

#6: Don’t Sweat the AWA Too Much

The final of my GMAT analytical writing tips is to not worry about the AWA section too much. As I mentioned in a previous section, the AWA section isn’t that important in the overall scheme of your GMAT score. It’d be a mistake to spend a lot of time and energy stressing over and preparing for the AWA section before you take the GMAT.

Spend between three to six hours preparing for the AWA, depending on how comfortable you are writing to the AWA’s structure. More often than not, that’s all the time test-takers need to achieve a solid AWA score.

Your GMAT AWA score won’t make or break your chance of admission to the business school of your dreams. An AWA score between 4-6 will sufficiently demonstrate your writing abilities to most admissions committees, and there’s not a huge advantage to scoring a perfect 6 on the AWA section.

An AWA score of below 4, however, will raise red flags for admissions committees who may question your communication abilities. So, it’s important to study for the AWA section to make sure your score is sufficient.

What’s Next?

Feeling set on GMAT analytical writing tips, but looking for more advice on other sections of the GMAT? We’ve got tons of in-depth, high-quality guides to help you master the content you’ll see on GMAT test day. Check out our guide to the GMAT verbal section to learn how to master the three GMAT question types or read our guide to the GMAT quant section  to understand exactly what math you need to know to achieve your goal GMAT score.

Looking to make an in-depth, comprehensive GMAT study plan? Our guide to GMAT study plans provides four sample study plans that you can adapt to your needs. Pick and choose between one-month, three-month, and six-month study plans that are each designed to boost your GMAT score.

Setting a realistic goal score is a hugely important part of your GMAT prep. By setting a realistic goal score, you give yourself a target to work towards and a benchmark by which to measure your progress as you prep for the GMAT. In our guide to GMAT score requirements, you’ll learn about how to set a goal that makes sense for your abilities and needs as a test-taker.

Was this helpful? Sign up for FREE GMAT and MBA guides!

Share this:.

  • Click to share on Twitter (Opens in new window)
  • Click to share on Facebook (Opens in new window)
  • Click to share on Google+ (Opens in new window)

gmat example essay prompts

Author: Hayley Milliman

Hayley Milliman is a former teacher turned writer who blogs about education, history, and technology. When she was a teacher, Hayley's students regularly scored in the 99th percentile thanks to her passion for making topics digestible and accessible. In addition to her work for PrepScholar, Hayley is the author of Museum Hack's Guide to History's Fiercest Females. View all posts by Hayley Milliman

Enjoy this post? Rate it!

GMAT essay topics

GMAT essay topics | 3 examples included to ace the exam!

The gmat essay topics are distinct from most essays you’ve written in college. it involves critical thinking skills, how well you articulate the matter without picking a side, and so on. read on to know more, table of contents, all you need to know about the gmat essay section, gmat essay topics | things to keep in mind, example 1: it is an excerpt from the grocery store’s business plan, example 2: the publisher of the mercury, a weekly newspaper, issued the following announcement, example 3: protesters at waymarsh state college, #1. time your gmat essay practice, #2. write your gmat essays, #3. assess the quality of your gmat essays, key takeaways.

The GMAT essay section provides a one-paragraph prompt containing a specific argument. This document includes practice questions to help you improve your essay section , help you analyze how well you have reasoned the statement, and so on. While attempting the essay section, you have to keep certain aspects in mind. To help you nail the exam, here are some GMAT essay topics that you must practice positively. 

Before proceeding ahead with the GMAT essay topics, here are a few things you need to keep in mind –

  • Ensure to analyze the argument’s logic and the incorporation of proof in the assertion during your discussion.
  • Consider the debatable assumptions that underpin thoughts and the possible theories or counterpoints that might undermine the conclusion. 
  • You can also talk about what kind of evidence would help or hurt the argument. 
  • Additionally, consider what changes to the debate would make it more logical and what would help you help assess its outcome.

GMAT essay topics |Examples that you must know of

Now that we’ve covered what the essay task requires, let’s first look at some sample GMAT essay topics from the official list.

No clear categories of provokes affect your assessment , but for a representative sample. It’s worth noting that the construction of each assertion does not fall within these boundaries; political prompts can employ the same flawed confrontational strategies as business or health and science prompts, for example.

Practicing GMAT essay topics before the actual exam

‘Even though we introduced a pharmacy segment to our supermarket this year, our overall sales have risen by 20%. The main concern of the customer is the convenience provided by one-stop shopping. Adding a clothing department and a vehicle supplies and repair shop is the most certain way to boost our profits in the coming years. In the coming years, we must also plan to add different departments and assistance, including a restaurant and a landscape shop. We will have a competitive edge over other local retailers since we are the only store in the district that offers such a wide range of services.’

Start writing answers wherein you discuss one or more clashing framework explanations and explain how the summaries can compensate for the proof stated in the proposition. Finally, look for logical flaws in the ending, facts, and rationale.

GMAT essay topics | 3 examples included to ace the exam!

‘Since the launch of a competing lower-priced newspaper, The Bugle, five years ago, The Mercury’s circulation has dropped by 10,000 readers.” The best way to encourage more people to read The Mercury is to lower its price less than that of the Bugle until circulation returns to pre-recession levels. Mercury’s increased circulation will entice more businesses to purchase ad space in the paper.’

Attempt writing responses in which you deliberate one or more contending framework explanations and describe how your descriptions can account for the evidence stated in the argument. Finally, examine the conclusion, evidence, and reasoning for logical flaws.

The following appeared in a local newspaper’s editorial section:

‘This past winter, 200 Waymarsh State College students marched to the state’s capital to hold rallies against proposed budget cuts to various state college programs.  On the other hand, the other 12,000 Waymarsh students either remained on campus or left for the winter holidays, indicating that they were less concerned with their education. Because the non-protesters outnumber the protesters, they represent the state’s college students more. As a result, the state legislature should disregard the protesting students’ pleas.’

Start to write solutions in which you clarify a few conflicting arrangement reasons and illustrate how the overviews can make up the difference for the verification outlined in the hypothesis. Finally, take a glance for technical inaccuracies in the conclusion, factual information, and reasoning skills.

GMAT essay topics | Tips to ace the section!

The tips that follow will assist you in preparing for any GMAT essay topics you may encounter on test day –

When practicing GMAT essay topics, remember that you have a strict 30-minute time limit for your essay.

You should start writing all of your practice essays on a compute r to simulate the situations of the computer-based GMAT.

After you’ve completed your practice GMAT essays, try to grade them using the 6-point grading set of guidelines and by correlating your writing to the GMAC’s sample essay.

  • The GMAT essay is unlike most of the essays you’ve written in college. 
  • The GMAC provides a list of the most formal GMAT essay topics asked in the exam. 
  • Ensure to practice the essay topics every day. 
  • The more you practice, the better you get at it. 
  • Answer mock GMAT essays. It will help you grasp the technique of writing excellent essays. 
  • Ensure that you articulate the matter well. Use your critical thinking skills for the same. 

If you need assistance in your GMAT preparation, we are here for you. Get in touch with our experts today!

Liked this blog? Then read the blog on the GMAT score chart | 5 points to enhance your exam strategy!

Question 1: What happens if a student fails the GMAT?

Answer: A student can take the GMAT up to five times yearly. As a result, even if you fail once, you will be given another chance.

Question 2: What is the most effective method for preparing for the GMAT?

Answer: The most effective way to prepare for the GMAT is to continue attempting and solving practice papers. We also recommend that you thoroughly examine the most common GMAT questions.

Question 3: Can I find the GMAT essay topics for practice online?

Answer: You can find the GMAT essay topics for practice online. As a matter of fact, you can visit the GMAC site for mock papers. 

How useful was this post?

Click on a star to rate it!

Average rating 0 / 5. Vote count: 0

No votes so far! Be the first to rate this post.

gmat example essay prompts

People also liked

IELTS success

IELTS success| Tips to master each section

IELTS Grammar

Importance of IELTS grammar | Tips and tricks

TOEFL and IELTS

TOEFL and IELTS | Role of contextual & academic words

simulated tests

Simulated tests | Opportunity to overcome challenges

GRE exam structure

TOEFL, IELTS, & GRE exam structure | Key differences 2024!

GMAT exam

Ace the GMAT exam | Navigate quant & verbal difficulties!

Leave a reply cancel reply.

Your email address will not be published. Required fields are marked *

Please enter an answer in digits: 13 + 6 =

Start your journey with iSchoolPrep

Need help with your Test Preparations? Contact Us for more details

gmat example essay prompts

Inquire Now

Get e-books, expert guidance, live classes and more....

Exams Know-how

GMAT AWA Essay ...

GMAT AWA Essay Topics: Know About GMAT Essay Topics

Author_Image

The Graduate Management Admission Test, or GMAT, is required for admission to business schools. The GMAT tests your understanding of basic arithmetic, algebra, geometry, multi-source data processing, and grammar.

The GMAT is divided into four sections: Analytical Writing Evaluation section, Integrated Reasoning section, Quantitative section, and Verbal section.

The GMAT AWA part of the GMAT exam assesses your analytical writing and critical-reasoning abilities. And, the best way to excel in this section is to practice rigorously.

In this blog, we will discuss the various GMAT essay topics and how to handle these GMAT writing prompts.

GMAT Analytical Writing Section

Listed below are a few facts about GMAT AWA essay section:

  • There are a variety of GMAT AWA topics in the GMAT exam out of which you are expected to compose an analytical essay by crtiquing the rationale of an argument.
  • You will be graded on how well you evaluate the reason and evidence for the argument, defend your thoughts, and structure your answer. You are given 30 minutes in this section to assess an argument.
  • The AWA score is not included in the overall GMAT score of 800. It is a different score in which the test-taker earns points from 0 to 6 in half-point scores.
  • Your essay will be evaluated and graded by both a computer and the examiner. You will get your AWA score, along with GMAT sores, within 15 days of taking the test.
  • All education institutions have cut-offs for the GMAT score out of 800, but the AWA score is out of 6. The cut-off range for the GMAT AWA section is between 4 to 4.5 for admission into universities.

GMAT analytical writing topics are marked based on the following skills:

Suggested: How to Prepare for GMAT AWA Section?

Presentation and Organization of the Essay

You will be marked based on how you portray your ideas while evaluating your work. If you have natural flows between concepts and a defined opening, body, and closing, your essay will be regarded as meaningful. However, your score will decline if your ideas are disorganised and erratic.

Linguistic Abilities

Linguistic abilities cover a variety of topics. The examiner assesses your technical command of conventional written English elements. The examiner will also evaluate whether your writing style contributes to a compelling or incorrect argument.

Suggested: GMAT Waiver 2022

Analytical Reasoning

Examiners will assess the quality of the notions and assertions you put forward when evaluating the level of your logical reasoning. They will also consider if your instances and relevant facts are pertinent to the case you are expressing.

There are various GMAT essay topics, let’s further look into what kinds of topics are asked in the AWA section of the GMAT exam:

GMAT AWA Topics

GMAT AWA prompts do not necessitate any prior understanding or prior knowledge of a certain field. They address a wide range of topics, including economics, politics, leadership, education, social issues, marketing, and the environment.

Here are a few sample GMAT essay prompts to help you guide how to write AWA in GMAT :

“This past winter, 200 students from Waymarsh State College travelled to the state capitol building to protest against proposed cuts in funding for various state college programs. The other 12,000 Waymarsh students weren’t so concerned about their education: they either stayed on campus or left for winter break. Since the group who did not protest is far more numerous, it is more representative of the state’s college students than are the protesters. Therefore, the state legislature need not heed the appeals of the protesting students.”

"While trucking companies that deliver goods pay only a portion of highway maintenance costs and no property tax on the highways they use, railways spend billions per year maintaining and upgrading their facilities. The government should lower the railroad companies’ property taxes, since sending goods by rail is a more appropriate mode of ground transportation than highway shipping. For one thing, trains consume only a third of the fuel a truck would use to carry the same load, making them a more cost-effective and environmentally sound mode of transport. Furthermore, since rail lines already exist, increases in rail traffic would not require building new lines at the expense of taxpaying citizens."

Suggested: GRE AWA Template

“Our total sales have increased this year by 20 percent since we added a pharmacy section to our grocery store. Clearly, the customer’s main concern is the convenience afforded by one-stop shopping. The surest way to increase our profits over the next couple of years, therefore, is to add a clothing department along with an automotive supplies and repair shop. We should also plan to continue adding new departments and services, such as a restaurant and a garden shop, in subsequent years. Being the only store in the area that offers such a range of services will give us a competitive advantage over other local stores.”

Question: Discuss how well reasoned you find this argument. In your discussion, be sure to analyze the line of reasoning and the use of evidence in the argument. For example, you may need to consider what questionable assumptions underlie the thinking and what alternative explanations or counterexamples might weaken the conclusion. You can also discuss what sort of evidence would strengthen or refute the argument, what changes in the argument would make it more logically sound, and what, if anything, would help you better evaluate its conclusion.

Suggested: GMAT AWA Template

Practising for the AWA section of the GMAT exam keeping in mind the kind of topics mentioned above will help you a great deal. You must begin your GMAT preparation in advance to obtain the necessary knowledge and confidence to perform well on the test. If you require further guidance and assistance, sign up for our Yocket Premium services now!

Frequently Asked Questions About GMAT AWA Topics

What is the score range of the AWA section?

The score range of the AWA section is between 0-6. 6 is outstanding, 5 is very good, 4 is good, 3 is adequate, 2 is less than adequate, 1 is poor, and 0 is no substantive response given.

What is the time duration of this section?

This section is 30 minutes long.

What are the topics included in the AWA section?

The topics asked in this section are Economics, Politics, Leadership, Education, Social Issues, Marketing, and the Environment.

How many points does one need in the AWA section to get into a good college?

Around 4-4.5 points are needed to get into a good college.

Is there a separate cut-off for the AWA section?

Yes, the cut-off for AWA section is separate from the general GMAT cut-off.  The cut-off for the AWA section is between 4-4.5 points for good colleges.

editor-picture

Kashyap Matani

Articles you might like

gmat example essay prompts

How to write your GMAT AWA Essay [Effectively]

gmat example essay prompts

INTRODUCTION

Hello there!

If you’ve found your way to this blog, we assume that you have begun to think about that often-neglected section of the GMAT – the AWA!

If wishes could come true, we’re guessing that GMAT aspirants would have wished away the AWA section away by now!

The AWA involves staring at a blank page and cursor and thinking up stuff to write, not an easy task for many people.

It also consumes a lot of mental bandwidth at the very beginning of the test.

To top it off, it does not even add to your final score!

We can understand why it isn’t your favorite section on the GMAT 🙂

But here’s the good news! There are not many shortcuts to mastering GMAT Quant and Verbal, but there are shortcuts to cracking the AWA section.

By the time you finish reading this guide, you will know what these hacks are. You will learn how to write an effective AWA essay that gets you a good score and leaves you charged for the real test that lies ahead.

This blog will teach you –

  • What you need to know BEFORE you start preparing for the AWA
  • How to use the 30 minutes allotted to AWA to maximum effect
  • How to use a template to make the AWA writing process simpler

Besides this, you will find 8 sample AWA essays to observe and learn from.

Happy reading! 🙂

7 Things to remember before you start prepping for the AWA

gmat example essay prompts

Before you dive into AWA preparation, there are a few things you should know about the AWA. Many of these facts will ease your AWA fears and bring a smile to your face!

1. Why is the AWA section on the GMAT?

Each section of the GMAT is carefully constructed ( at the expense of millions of dollars, we kid you not!), to test your readiness for an MBA program and for your post-MBA career. One of the skills you will definitely need post-MBA is an ability to analyze an argument impartially and convey your perspective clearly.

This is what the AWA tests you on.

2. On the AWA, you need to be a lawyer, that is, you need to find faults with the given argument. You do not need to be a journalist, that is, you do not need to write about all possible perspectives of an issue.

Also remember, that this is an analysis, not an opinion piece. Do not bring YOUR perspective and your opinions into the essay. Your only goal is to analyse the given argument.

3. Unlike the Quant and Verbal sections, where your thumb-rule should be to get as high a score as possible, we’d suggest that you do not expend too much mental energy on the AWA, trying to score a 6 on 6.

Getting a perfect 6 will look good on your GMAT score card, and will sound great as you’re telling your friends about it. But it will not be the make-or-break factor in your application. A 4 or a 5 is good enough.

4. The AWA is graded by an E-reader application and by a human reader. Since there is an element of automated grading, you can rig the test to an extent. We’ve found that if you write a substantial essay of over 500 words, and if you structure the argument well ( check our CrackVerbal template in the following chapter), you are almost guaranteed to get a 4+ score!

5. Assuming that you prepare for the GMAT over a course of three months, we recommend that you practice writing 5 to 10 essays, and make sure you get feedback for all of them. If you cover this much practice ground, you’re good to go!

6. One of the best things about the AWA section is that you know all of the questions beforehand (yes, they’re all up there on the GMAC site – Analytical Writing Section ). So you do not have to go hunting for ‘authentic’ AWA essay questions.

7. Now you can choose the order in which you want to take up the sections before starting the test. It is advisable to keep in mind the order that would be helpful for you and prepare for the AWA based on that strategy. This is a recent change to the GMAT test structure. It was introduced in July 2017. We have done a detailed analysis of what this means to an Indian GMAT test-taker in the this blog

8. We saved the coolest point for last 🙂

The AWA lends itself very easily to the use of an essay template. No matter what the argument prompt is, you can bet that there will be at least 3 glaring errors of logic in it. You can, therefore, use a template to structure your AWA essay. Using a template takes most of the stress away from the AWA section.

In the few minutes before you start, you can jot down the template on your scratchpad, so that you don’t have to remember it anymore. Also, because you can plan many of your sentences beforehand, you can get at least a 100 words down before you even read the question!

There are a lot of templates on the internet – probably the most famous one being the Chineseburned AWA template.

At CrackVerbal, we have our own template for the AWA, a modified version of the Chineseburned template. We call it the CrackVerbal AWA Template on Steroids! 🙂

The AWA Writing Process

MBA Essay Writing

1. Write your templatized response

This should take you about 5 minutes:

Type out your prepared template response. Below is a sample. We definitely do not recommend that you use the same words. What you can do, however, is read a few templates on the net, and then write your own. Since you have written it yourself, it will be that much easier to memorise it.

CRACKVERBAL AWA TEMPLATE

The argument claims that < restate the argument >. Stated in this way the argument fails to take into account a few key factors which could call the conclusion to question. It rests on some assumptions, for which there is no clear evidence. Therefore, the argument is unconvincing and falls apart at the seams.

Paragraph 1:

1. Firstly, ( ) 2. This statement is a stretch and not substantiated in any way. 3. The argument would have been much clearer if ( )

Paragraph 2:

1. Second ( ). This is again a very weak and unsupported claim as the argument ( ). 2. For example, 3. This argument would have sounded a lot more convincing if 4. In addition, it would have been strengthened ever further if the argument provided evidence that

Paragraph 3:

1. Finally, the argument concludes that 2. However, what is not clear here is ( ) 3. If there had been evidence to support ( )

In summary, the argument fails to convince because of the faulty assumptions aforementioned. If the argument had drawn upon examples as suggested, and thereby plugged in the holes in the reasoning, it would have been far sounder on the whole.

2. Brainstorm

Now that you have put the pre-planned portion of the essay down, it’s time to read the AWA prompt and wear your thinking hat. GMAT, in its politically correct, non-partisan way, says ‘Discuss how well-reasoned you find this argument’. Remember however, that an AWA argument is never well-reasoned!

There are always a couple of glaring flaws in logic you can pounce on. If these flaws do not occur to you immediately, because of test-day stress, do not assume that you have been given a particularly sound argument. There is no such thing on the AWA!

If you’re unable to be critical, imagine that the author of the argument is somebody you dislike..a teacher you hated at college, or that guy who overtook you and almost dented your car this morning! There, now you’re in the right frame of mind to attack the argument 🙂

Before you do so, you need to understand the three elements of the argument – Conclusion, Premise and Assumptions.

Let us look at an example, and detect these three elements.

“Most companies would agree that as the risk of physical injury occurring on the job increases, the wages paid to employees should also increase. Hence it makes financial sense for employers to make the workplace safer: they could thus reduce their payroll expenses and save money.”

The conclusion is the decision/statement that the author has arrived at. In this case, the conclusion is the last sentence – “Hence it makes financial sense for employers to make the workplace safer: they could thus reduce their payroll expenses and save money.”

The premises are the building blocks of facts on which the conclusion rests. In other words, a premise is what is offered as support for the conclusion. In this case, the premise is – Most companies would agree that as the risk of physical injury occurring on the job increases, the wages paid to employees should also increase.

Assumptions are the unstated, unwritten premises that plug the gap between the written premises and the conclusion. It is the assumptions that you need to attack on the AWA!

How do you attack assumptions?

Remember that an assumption can be incorrect for a variety of reasons. Here are a few types of incorrect assumptions –

1. The Sampling Assumption – The sampling argument assumes that a small group is representative of a much larger group to which it belongs.

2. The illogical analogy assumption – The illogical analogy states that because something applies to A, it applies to B also.

3. The Causal Assumption – The Causal Assumption confuses correlation with causation. That means, just because ‘A’ usually occurs after ‘B’ occurs, does not necessarily imply that B happens because of A.

4. The Data Bias – This occurs when the data for a statistical inference itself is drawn from a sample that is not representative of the population under consideration. This is a case of faulty data leading to faulty assumptions.

5. The Non Sequitur – This simply means, finding a connection where there is none. Non Sequitur means “does not follow,” which is short for: the conclusion does not follow from the premise.

Don’t let these categories overwhelm you. We’ve put them down here to get you thinking. However, you can find faulty assumptions with ease, even if you have no clue what a non sequitur is!

As you brainstorm, you will need to jot down your thoughts on the scratchpad. Keep it crisp and brief. Make sure you have these things down –

1. Conclusion + Premise: 2. Flawed Assumption #1: 3. Flawed Assumption #2: 4. Flawed Assumption #3:

For each assumption, also make a cursory note of why it is flawed, an example that talks about why it is flawed, and what additional data would strengthen the argument ( or if you are convinced that you can remember these additional details without having to make a note of them, you can get on with the writing! )

This should take you about 15 minutes:

Here is where you fill in your templatized response with specific details.

The only detail you need to add to the first paragraph is a summary of the argument that is presented. In the above template, your summary should go here ->

1. Start off by pointing out the first flawed assumption. 2. Explain why this assumption is flawed. 3. Give an example that supports the flaw. 4. Explain what further information could have strengthened this argument.

1. Start off by pointing out the second flawed assumption. 2. Explain why this assumption is flawed. 3. Give an example that supports the flaw. 4. Explain what further information could have strengthened this argument.

Paragraph 4:

1. Start off by pointing out the third flawed assumption. 2. Explain why this assumption is flawed. 3. Give an example that supports the flaw. 4. Explain what further information could have strengthened this argument.

Paragraph 5:

This is the concluding paragraph. You already have it down in your template! 🙂

4. Proofread

Are you wondering if three minutes is really enough time to proof-read a 500 word essay?

Here’s the deal – The AWA section is about whether you can analyse an argument and discuss it in an articulate manner. It is not a test of grammar and spelling. Hence, the GMAT will excuse minor errors in spelling and grammar.

However, you should understand that a human reader is going to be reviewing your work, and any human reader will have an unconscious bias against bad grammar and spellings. Hence, you want to keep your essay as error-free as possible, without worrying about it too much.

Three minutes should be able time for you to quickly glance through the document and make sure you haven’t made any obvious errors.

Voila! 🙂 Your AWA essay is ready!

Also Read: GMAT Section Selection – Everything you need to know

Sample AWA Essays

gmat example essay prompts

Sample Essay 1

”Most companies would agree that as the risk of physical injury occurring on the job increases, the wages paid to employees should also increase. Hence it makes financial sense for employers to make the workplace safer: they could thus reduce their payroll expenses and save money.”

Discuss how well reasoned you find this argument. In your discussion be sure to analyze the line of reasoning and the use of evidence in the argument.

For example, you may need to consider what questionable assumptions underlying the thinking and what alternative explanations or counter examples might weaken the conclusion. You can also discuss what sort of evidence would strengthen or refute the argument, what changes in the argument would make it more logically sound, and what, if anything, would help you better evaluate its conclusion.

Introduction:

This argument states that it makes financial sense for employers to make the workplace safer because lower wages could then be paid to employees. This conclusion is based on the premise that as the risk of physical injury increases, the wages paid to employees should also increase. However, this argument makes several unsupported assumptions. For example, the argument assumes that the costs associated with making the workplace safe do not outweigh the increased payroll expenses due to hazardous conditions.

Body Paragraph 1

The first issue to be addressed is whether increased labor costs justify large capital expenditures to improve the work environment. Clearly one could argue that if making the workplace safe would cost an exorbitant amount of money in comparison to leaving the workplace as is and paying slightly increased wages than it would not make sense to improve the work environment. For example, if making the workplace safe would cost $100 million versus additional payroll expenses of only $5,000 per year, it would make financial sense to simply pay the increased wages. No business or business owner would pay all that extra money just to save a couple dollars and improve employee health and relations. To consider this, a cost benefit analysis must be made. I also feel that although a cost benefit analysis should be the determining factor with regard to these decisions making financial sense, it may not be the determining factor with regard to making social, moral and ethical sense.

Body Paragraph 2

Finally one must understand that not all work environments can be made safer. For example, in the case of coal mining, a company only has limited ways of making the work environment safe. While companies may be able to ensure some safety precautions, they may not be able to provide all the safety measures necessary. In other words, a mining company has limited ability to control the air quality within a coal mine and therefore it cannot control the risk of employees getting black. In other words, regardless of the intent of the company, some jobs are simply dangerous in nature.

In conclusion, while at first it may seem to make financial sense to improve the safety of the work environment sometimes it truly does not make financial sense. Furthermore, financial sense may not be the only issue a company faces. Other types of analyses must be made such as the social ramifications of an unsafe work environment and the overall ability of a company to improve that environment (i.e., coal mine). Before any decision is made, all this things must be considered, not simply the reduction of payroll expenses.

Sample Essay 2

The following appeared in a memorandum issued by a large city’s council on the arts.

“In a recent citywide poll, fifteen percent more residents said that they watch television programs about the visual arts than was the case in a poll conducted five years ago. During these past five years, the number of people visiting our city’s art museums has increased by a similar percentage. Since the corporate funding that supports public television, where most of the visual arts programs appear, is now being threatened with severe cuts, we can expect that attendance at our city’s art museums will also start to decrease. Thus some of the city’s funds for supporting the arts should be reallocated to public television.”

Discuss how well reasoned… etc.

Introduction

In this argument the author concludes that the city should allocate some of its arts funding to public television. The conclusion is based on two facts: (1) attendance at the city’s art museum has increased proportionally with the increases in visual-arts program viewing on public television, and (2) public television is being threatened by severe cuts in corporate funding. While this argument is somewhat convincing, a few concerns need to be addressed.

To begin with, the argument depends on the assumption that increased exposure to the visual arts on television, mainly public television, has caused a similar increase in local art-museum attendance. However, just because increased art-museum attendance can be statistically correlated with similar increases in television viewing of visual-arts programs, this does not necessarily mean that the increased television viewing of arts is the cause of the rise in museum attendance.

Moreover, perhaps there are other factors relevant to increased interest in the local art museum; for instance, maybe a new director had procured more interesting, exciting acquisitions and exhibits during the period when museum attendance increased, in addition, the author could be overlooking a common cause of both increases. It is possible that some larger social or cultural phenomenon is responsible for greater public interest in both television arts programming and municipal art museums.

Body Paragraph 3

To be fair, however, we must recognize that the author’s assumption is a special case of a more general one that television viewing affects people’s attitudes and behavior. Common sense and observation tell me that this is indeed the case. After all, advertisers spend billions of dollars on television ad time because they trust this assumption as well.

In conclusion, I am somewhat persuaded by this author’s line of reasoning. The argument would be strengthened if the author were to consider and rule out other significant factors that might have caused the increase in visits to the local art museum.

Sample Essay 3

The following appeared in a report presented for discussion at a meeting of the directors of a company that manufactures parts for heavy machinery.

“The falling revenues that the company is experiencing coincide with delays in manufacturing. These delays, in turn, are due in large part to poor planning in purchasing metals. Consider further that the manager of the department that handles purchasing of raw materials has an excellent background in general business, psychology, and sociology, but knows little about the properties of metals. The company should, therefore, move the purchasing manager to the sales department and bring in a scientist from the research division to be manager of the purchasing department.”

In response to a coincidence between falling revenues and delays in manufacturing, the report recommends replacing the manager of the purchasing department. The grounds for this action are twofold. First, the delays are traced to poor planning in purchasing metals. Second, the purchasing manager’s lack of knowledge of the properties of metals is thought to be the cause of the poor planning. It is further recommended that the position of the purchasing manager be filled by a scientist from the research division and that the current purchasing manager be reassigned to the sales department. In support of this latter recommendation, the report states that the current purchasing manager’s background in general business, psychology, and sociology equip him for this new assignment. The recommendations advanced in the report are questionable for two reasons.

To begin with, the report fails to establish a causal connection between the falling revenues of the company and the delays in manufacturing. The mere fact that falling revenues coincide with delays in manufacturing is insufficient to conclude that the delays caused the decline in revenue. Without compelling evidence to support the causal connection between these two events, the report’s recommendations are not worthy of consideration.

Second, a central assumption of the report is that knowledge of the properties of metals is necessary for planning in purchasing metals. No evidence is stated in the report to support this crucial assumption. Moreover, it is not obvious that such knowledge would be required to perform this task. Since planning is essentially a logistical function, it is doubtful that in-depth knowledge of the properties of metals would be helpful in accomplishing this task.

In conclusion, this is a weak argument. To strengthen the recommendation that the manager of the purchasing department be replaced, the author would have to demonstrate that the falling revenues were a result of the delays in manufacturing. Additionally, the author would have to show that knowledge of the properties of metals is a prerequisite for planning in purchasing metals.

Sample Essay 4

The following appeared in an announcement issued by the publisher of The Mercury, a weekly newspaper.

“Since a competing lower-priced newspaper, The Bugle, was started five years ago, The Mercury’s circulation has declined by 10,000 readers. The best way to get more people to read The Mercury is to reduce its price below that of The Bugle, at least until circulation increases to former levels. The increased circulation of The Mercury will attract more businesses to buy advertising space in the paper.”

A newspaper publisher is recommending that the price of its paper, The Mercury, be reduced below the price of a competing newspaper, The Bugle. This recommendation responds to a severe decline in circulation of The Mercury during the 5-year period following the introduction of The Bugle. The publisher’s line of reasoning is that lowering the price of The Mercury will increase its readership, thereby increasing profits because a wider readership attracts more advertisers. This line of reasoning is problematic in two critical respects.

While it is clear that increased circulation would make the paper more attractive to potential advertisers, it is not obvious that lowering the subscription price is the most effective way to gain new readers. The publisher assumes that price is the only factor that caused the decline in readership. But no evidence is given to support this claim. Moreover, given that The Mercury was the established local paper, it is unlikely that such a mass exodus of its readers would be explained by subscription price alone.

There are many other factors that might account for a decline in The Mercury’s popularity. For instance, readers might be displeased with the extent and accuracy of its news reporting, or the balance of local to other news coverage. Moreover, it is possible The Mercury has recently changed editors, giving the paper a locally unpopular political perspective. Or perhaps readers are unhappy with the paper’s format, the timeliness of its feature articles, its comics or advice columns, the extent and accuracy of its local event calendar, or its rate of errors.

In conclusion, this argument is weak because it depends on an oversimplified assumption about the causal connection between the price of the paper and its popularity. To strengthen the argument, the author must identify and explore relevant factors beyond cost before concluding that lowering subscription prices will increase circulation and, thereby, increase advertising revenues.

Sample Essay 5

The following appeared as part of an article in a magazine devoted to regional life.

“Corporations should look to the city of Helios when seeking new business opportunities or a new location. Even in the recent recession, Helios’s unemployment rate was lower than the regional average. It is the industrial center of the region, and historically it has provided more than its share of the region’s manufacturing jobs. In addition, Helios is attempting to expand its economic base by attracting companies that focus on research and development of innovative technologies.”

In this argument corporations are urged to consider the city of Helios when seeking a new location or new business opportunities. To support this recommendation, the author points out that Helios is the industrial center of the region, providing most of the region’s manufacturing jobs and enjoying a lower-than-average unemployment rate. Moreover, it is argued, efforts are currently underway to expand the economic base of the city by attracting companies that focus on research and development of innovative technologies. This argument is problematic for two reasons.

To begin with, it is questionable whether the available labor pool in Helios could support all types of corporations. Given that Helios has attracted mainly industrial and manufacturing companies in the past, it is unlikely that the local pool of prospective employees would be suitable for corporations of other types. For example, the needs of research and development companies would not be met by a labor force trained in manufacturing skills. For this reason, it’s unlikely that Helios will be successful in its attempt to attract companies that focus or research and development of innovative technologies.

Another problem with the available work force is its size. Due to the lower than average unemployment rate in Helios, corporations that require large numbers of workers would not find Helios attractive. The fact that few persons are out of work suggests that new corporations will have to either attract new workers to Helios or pay the existing workers higher wages in order to lure them away from their current jobs. Neither of these alternatives seems enticing to companies seeking to relocate.

In conclusion, the author has not succeeded in providing compelling reasons for selecting Helios as the site for a company wishing to relocate. In fact, the reasons offered function better as reasons for not relocating to Helios. Nor has the author provided compelling reasons for companies seeking new business opportunities to choose Helios.

Sample Essay 6

The following appeared in the health section of a magazine on trends and lifestyles.

“People who use the artificial sweetener aspartame are better off consuming sugar, since aspartame can actually contribute to weight gain rather than weight loss. For example, high levels of aspartame have been shown to trigger a craving for food bydepleting the brain of a chemical that registers satiety, or the sense of being full. Furthermore, studies suggest that sugars, if consumed after at least 45 minutes of continuous exercise, actually enhance the body’s ability to burn fat. Consequently, those who drink aspartame-sweetened juices after exercise will also lose this calorie-burning benefit. Thus it appears that people consuming aspartame rather than sugar are unlikely to achieve their dietary goals.”

In this argument the author concludes that people trying to lose weight are better off consuming sugar than the artificial sweetener aspartame. To support this conclusion the author argues that aspartame can cause weight gain by triggering food cravings, whereas sugar actually enhances the body’s ability to burn fat. Neither of these reasons provides sufficient support for the conclusion.

The first reason that aspartame encourages food cravings is supported by research findings that high levels of aspartame deplete the brain chemical responsible for registering a sense of being satedHidden text (sated, sating ), or full. But the author’s generalization based on this research is unreliable. The research was based on a sample in which large amounts of aspartame were administered; however, the author applies the research findings to a target population that includes all aspartame users, many of whom would probably not consume high levels of the artificial sweetener.

The second reason that sugar enhances the body’s ability to burn fat is based on the studies in which experimental groups, whose members consumed sugar after at least 45 minutes of continuous exercise, showed increased rates of fat burning. The author’s general claim, however, applies to all dieters who use sugar instead of aspartame, not just to those who use sugar after long periods of exercise. Once again, the author’s generalization is unreliable because it is based on a sample that clearly does not represent all dieters.

To conclude, each of the studies cited by the author bases its findings on evidence that does not represent dieters in general; for this reason, neither premise of this argument is a reliable generalization. Consequently, I am not convinced that dieters are better off consuming sugar instead of aspartame.

Sample Essay 7

The following appeared in the editorial section of a corporate newsletter.

“The common notion that workers are generally apathetic about management issues is false, or at least outdated: a recently published survey indicates that 79 percent of the nearly 1,200 workers who responded to survey questionnaires expressed a high level of interest in the topics of corporate restructuring and redesign of benefits programs.”

Based upon a survey among workers that indicates a high level of interest in the topics of corporate restructuring and redesign of benefits programs, the author concludes that workers are not apathetic about management issues. Specifically, it is argued that since 79 percent of the 1200 workers who responded to survey expressed interest in these topics, the notion that workers are apathetic about management issues is incorrect. The reasoning in this argument is problematic in several respects.

First, the statistics cited in the editorial may be misleading because the total number of workers employed by the corporation is not specified. For example, if the corporation employs 2000 workers, the fact that 79 percent of the nearly 1200 respondents showed interest in these topics provides strong support for the conclusion. On the other hand, if the corporation employs 200,000 workers, the conclusion is much weaker.

Another problem with the argument is that the respondents’ views are not necessarily representative of the views of the work force in general. For example, because the survey has to do with apathy, it makes sense that only less apathetic workers would respond to it, thereby distorting the overall picture of apathy among the work force. Without knowing how the survey was conducted, it is impossible to assess whether or not this is the case.

A third problem with the argument is that it makes a hasty generalization about the types of issues workers are interested in. It accords with common sense that workers would be interested in corporate restructuring and redesign of benefits programs, since these issues affect workers very directly. However, it is unfair to assume that workers would be similarly interested in other management issues—ones that do not affect them or affect them less directly.

In conclusion, this argument is not convincing as it stands. To strengthen it, the author would have to show that the respondents account for a significant and representative portion of all workers. Additionally, the author must provide evidence of workers’ interest other management topics—not just those that affect workers directly.

Sample Essay 8

The following appeared in the opinion column of a financial magazine.

“On average, middle-aged consumers devote 39 percent of their retail expenditure to department store products and services, while for younger consumers the average is only 25 percent. Since the number of middle-aged people will increase dramatically within the next decade, department stores can expect retail sales to increase significantly during that period. Furthermore, to take advantage of the trend, these stores should begin to replace some of those products intended to attract the younger consumer with products intended to attract the middle-aged consumer.”

Based on an expected increase in the number of middle-aged people during the next decade, the author predicts that retail sales at department stores will increase significantly over the next ten years. To bolster this prediction, the author cites statistics showing that middle-aged people devote a much higher percentage of their retail expenditure to department-store services and products than younger consumers do. Since the number of middle-aged consumers is on the rise and since they spend more than younger people on department-store goods and services, the author further recommends that department stores begin to adjust their inventories to capitalize on this trend. Specifically, it is recommended that department stores increase their inventory of products aimed at middle- aged consumers and decrease their inventory of products aimed at younger consumers. This argument is problematic for two reasons.

First, an increase in the number of middle-aged people does not necessarily portend an overall increase in department-store sales. It does so only on the assumption that other population groups will remain relatively constant. For example, if the expected increase in the number of middle-aged people is offset by an equally significant decrease in the number of younger people, there will be little or no net gain in sales.

Second, in recommending that department stores replace products intended to attract younger consumers with products more suitable to middle-aged consumers, the author assumes that the number of younger consumers will not also increase. Since a sizable increase in the population of younger consumers could conceivably offset the difference in the retail expenditure patterns of younger and middle- aged consumers, it would be unwise to make the recommended inventory adjustment lacking evidence to support this assumption.

This argument is unacceptable. To strengthen the argument the author would have to provide evidence that the population of younger consumers will remain relatively constant over the next decade.

We hope that our strategies help you conquer GMAT AWA with enough and more energy to spare for the sections that follow!

Now that you’ve figured out how to tackle the AWA section, do you want to put theory to practice and get your AWA essay graded?

Our experts here at CrackVerbal will evaluate and grade your AWA essay and give you specific, actionable feedback.

gmat example essay prompts

avatar

  • E-mail & Password
  • Notification Settings
  • Global Settings
  • Applicant profile
  • Update status
  • My GMAT info
  • --> My Education -->